Вы находитесь на странице: 1из 138
How To Solve It A New Aspect of Mathematical Method G. POLYA Stanford University Princeton University Press Princeton, New Jersey Copyright 1945 by Princeton University Press Copyright © renewed 1973 by Princeton University Press Second Euition Copyright © 1957 by G. Polya AIL Rights Reserved LO. Cand: 7960544 ISBN 0.691.02956-5 (paperback edn) ISHN 0-691-08097-6 (hardcover cde.) Fit Princeton Paperback Printing, 1971 Second Printing, 2975 This book is sold subject to the condition that it shall not, by way of trade, be lent, resold, hired out, or otherwise disposed of ‘without the publisher's consent, in any form of bind ing or cover other than that in which it iv published. Printed in the United States of America by Princeton University Press, Princeton, New Jersey From the Preface to the First Printing A great discovery solves a grent problem but there is a rain of discovery in the solution of any problem. Your [Problem may be modest: But if i challenges your eutiox ify and brings into play your inventive faculties, and if you solve i By your own means, you may experience the tension and enjoy the triumph of discovery. Such experi- fences at a susceptible age may cate a tase for mental ‘work and leave their imprint on mind and character for lifetime. Thus, a teacher of mathematics has a great epport nity. If he fills his alloted time with drilling his students in routine operations he kille chelr interes, hampers their intellectual development, and misuse hid oppor: tunity. But if he challenges the curiosity of tis students by setting them problems proportionate co their know! ‘edge, and helps them to solve their problema with stmu- lating questions, he may give them a taste for, and some smeans of, independent thinking ‘Alo a student whote college curriculum includes some mathematics bas a singular opportunicy. This oppor: nity is lost, of course, if be regards mathematics 26 a subject in which he has to earn x0 and so much eredit ‘and which he should forget alter the final examination au quickly as posible, The opportunity may be lost even if the student has tome natural talent for mathematics because he, 24 everybody else, must discover his talen's and tastes; he cannot know that he likes raspberry pie if be has never tasted raspberry pie, He may manage to find ‘out, however, that 2 mathematics problem may be as ‘much fun a3 erossword pezze, o that vigorous mental vi From the Prelace to the First Printing work may be an exercise as desirable at a fat game of tennis, Having tasted the pleasure in mathematic he will, not forget it easly and then there isa good chance that mathematics will becomne something, for him; a hobby, or 4 tool of his profesion, or his profession, ot a great ambition, ‘The author remembers the time when he wat a student ‘himself, a somewhat ambitious student, eager to und stand 2 lide mathematics and physics. He listened t0 leetures, reid books, tried to take in che solutions and facts presented, but tere was a question that dsrurbed Ihim again and again: "Yes, the sokution sews o work, it appears to be corset: but how is it possible to invent such 2 solution? Yes, this experiment stems to wok, this appears to be a fact; but how can people discover such facts? And how could I invent or discover sch things by myself?” Today the author is teaching mathematics in a ‘university; he thinks or hopes tha some of his mare cager students ask similar questions and he tiesto satisfy their curiosity. Trying to understand not only the solution of this or that problem but alo the mo:ives and procedures ‘of the solution, and trying to explain these motives and procedures to others, he was Enally Jed to write the present book. He hopes that it will be useful to teachers who wish to develop their students’ ability o solve prob- Jems, and ro students who ae keen on developing theit ‘own abilities. Although the present book pays special attention to the requirements of students and teachers of mathematics, it should interest anyboly concerned with the ways and ‘means of invention and discovery. Such interest may be more widespread than one would assume without rele tion, The space devoted by popular newspapers and magazines to crossword puzzles and other riddles seems tw show that people spend some vime in solving unprac From the Preface to the Fizst Printing il steal problems, Behind the desize to solve this or that problem that confers no material advantage, there may Fea deeper curiosity, adesive 19 understand the ways and the motiver and procedures, of solution. TThe following pages are writen somewbat concisely, Dat 2+ simply as possible, and are based on a Yong and serious study of methods of solution. This sort of study, talled hewrstie by some veriters, is not in fashion now. fadays but has a long past and, perhaps, some future. Studying the methods of solving problems, we perceive another face of mathematic. Yer, mathematics has two faces, it i the vigorous science of Euclid but it is also something cli, Mathematics prevented in the Budlidean sway appenrs 28a systematic, deductive seience; but mathe: mnaties in the making appears 2s an experimental, in ductive science, Both aspects ae as old asthe science of mathematics tell. But the second aspect is new in one respect; mathematics “in statu nascendi,” in the process fof being invented, has never before been presented in {quite this manner to the student, or to the teacher him: tell, of to the gencral public. The subject of heuristic has manifold connections; mathematicians, logicians, psychologists, educationalists, teen philosophers may clatn Various parts of i as belong fing to their special domains. The author, well aware of the posibilty of criticism from opposite quarters and Aeenly conscious of his Jimitations, bas one claim to make: he has some experience in solving problems and in teaching mathematics on various level “The subject is more fully dealt with in a more exten- sive book by the author which is on the way to com: pletion. Stanford University, August 5, 1044 From the Preface to the Seventh Printing From the Preface to the Seventh Printing ‘Lam glad to say that Thave now succeeded in falfiling, a least in pare, 2 promise given in the preface to the first princing: The two volumes Induction and Analogy in Mathematics and Patterns of Plausibie inference whieh ‘omsticuce my recemt work Mathematics and. Plansible Reasoning continue the line of thinking begun in How to Sotve Mt Zurich, August 30, 1954 Preface to the Second Biition ie Preface to the Second Fsition ‘The preient second edition adds, besides a few minor Smprovernents, 2 new fourth part, "Problems, Hints, Solutions.” ‘As this edition was being prepared for print, a study appeared (Educational Testing Service, Princeton, N.}: of. Time, June 38, 1936) which seems to have formu lated a fev pertinent observations=they are not new to the people in the knov, but it was high time to formu- late them forthe general public: mathematics has the dubious honor of being che lease popular subject ia the curriculum .. . Futare teachers pus through the elementary schools learning to detest mathematics ‘They return to the elementary scboal to teach a new generation to detest i T hope that the present edition, designed for wider difusion, will convince some of its seadars dhat mathe matics, besides being a necessary avenue to engineeting jobs and scientific knowiedge, may be fu and may 210 ‘open up a vista of mental activity on the highest level Zurich, Fune 30, 1956 Contents From the Preface 9 the First Printing From the Prelace to the Seventh Printing Preface to the Second Edicion. “How ‘Lo Solve It" list Introduction PART I, IN THE CLASSROOM Purpose ‘Helping the seudent 2. Questions, recommendations, mental operations Gonerality ‘Common sense ‘Teacher and student. imitation and practice te Mein divisions, main questions Four phases Understanding the problest Example Devising a plan 0, Example 31, Carrying out the plan aw xii Contents 22 Example Looking back Pxample Various approaches ‘The teacher's method of questioning Good questions and bad questions More examples 18. A problem of construction 19. A problem to prove 20. A rate problem PART II. HOW TO SOLVE IT A dialogue PART IM, SHORT DICTIONARY OF HEURISTIC Analogy ‘Ausiliary lements Auxiliary problem Bokano Bright idea (Can you check the result? Can you derive the result diferent? Can you use the result? Canrying out a3 4 6 5 % 9 38 2 46 se Ea 8 6 6 58 Contents Condition Contradicioryt Gorollary Gould you derive something useful from the data? Gould you restate the problem} Decorposing and recombining Definition Descartes Dete Diagnosis Dia you use all the data? Do you kaow a related problem? rave a figuret Examine your guess iguree Generalization Have you seen it before? ‘Here isa problem related to yours and solved before Rew Rewristic rezoning {Bt you cannot soe the propose problem veal fndetion ation, bope, success ion and mat “Aoventors paradox Fic possible to sats the condition? ibita Lerma “ oomtins niy rseatereec, 8 103 08 5 m4 a 195 125 xv Contents Look at the unknown Modern heuristic Notation Pappus ‘Pedantry and mastery Practical problems Problems to find, problems to prove Progress and achievement Puzrles ‘Reducto ad absurdum and indirect proof Redundant Routine problem Rules of discovery Rules of style Rules of teaching Separate the various parts of the condition Secting up equations Signs of progress Specialization Subconscious work Symmetry Terms, old and new Test by dimension “The future mathematician ‘The intelligent problem-solver ‘The intttigent reader ‘The traditional mathematics profesor ‘concn nly antec 123 19 134 Mat i M9 18 37 Contents Variation of the problem ‘What is the unknown? Why prools ‘Wisdom of proverbs Working backwards PART IV, PROBLEMS, HINTS, SOLUTIONS Problems Bins. Solutions 209 a4 251 258 242 How To Solve It fod 1291 08 ue 27 3 20 ‘aoe ® omen, GO AHL ONIAaKVASWIaNA LI FATOS OL MOH ine "wDIRord Dove sor May sun oF fa woqgond pasodond yp SMS ee RON pmege wenn nog sed sno go funy om. MORTEM mG UY ERD os honnag wont 397 pane “pues -aoygaud any pumpirpun 07 94 WOR Introduction ‘The following considerations are grouped around the preceding list of questions and suggestions entitled "How fo Solve It." Any question or suggestion quozed from it will be printed in italics, and the whole list will be referred to simply as “the lst" or a5 “our Hist The following pages will discuss the purpose of the lise, illustrate its practieal use by examples, and explain the underlying notions and mental operations. By way of prcliminary explanation, this much may be said: If ting dhem properly, you address these questions and suggestions to yourself, they may help you to solve your problem. Jf, using them properly, you address the same questions and suggestions to one of your students, you may help him co solve his problem. The book is divided into four parts. The ttle of the first part ie "Ta the Classroom." It contains twenty sections. Exch section will be quoted by its number im heavy type at, for instanee, “section 7." Sections 1 to 5 discuss the “Purpose” of our lst in gen- eral terms. Sections 6 to 17 explain what a Divisions, Main Questions” of the lit, and Practical example. Sections 18, 19, 20 add "More Ex- amples” ‘The title of the very short second part is “How to Solve 1. Te is written in dialogue; a somewhat idealized deacher answers short questions of a somewhat idealized sfudent . The third and most extensive part isa "Short Diction- of Heuristic"; we shall relzr to it asthe "Dietionan a Introduction 1 contains sixtyseven articles arranged alphabeticaly, For example, the meaning of the term steunientc. (set in small capitals) is explained in an article with this tite fon page 11a. When the title of such an article is referred to within the text ie will bese: in sovalleapitals. Cera paragraphs ofa few articles are more technical; they ate enclosed in square brackets. Some articles are fairly closely connected with the fist part to which they add further ilustrations and score specific comments. Other articles go somewhat beyond the aim of the fist part of whieh they explain the background. ‘There is 2 key: article on NoDIAN aezuausTic. 1c explains the connection ‘of the main articles and the plan underlying the Diction 217; ft containe also directions how to find information about particular items of the list. It must be emphasized that here i «common plan and a certain unity, becasse the articles of the Dictionary show the greatest outward variety. There ate a fev longer artiles devoted to the systematic though condensed discussion of some generat theme; others contain more specific comments, still others ross references, oF historical data, or quotations, or aphorisms, or even jokes. ‘The Dictionary should no: be read too quickly its text is often condensed, and now and then somewhat subsle- ‘The reader may reler to the Dictionary for information about particular points If these points come from hit experience with his own problems or his own students, the reading hat a much bevter chance to be profitable ‘The tle of Uhe fourth partis “Problems, Hfins, Solu tons." Te proposes a few problems to the more ambitious reader, Each problem is followed (in proper distance) by fa “hint” wast may reveal a way to the result which it explained inthe “tlution.” ‘We have mentioned repeatedly the “student” and the “teacher” and we thallzeler to and again. It Introduction at may be good to observe thatthe “student” may be a high shoo! student, or a college student, or anyone else who is studying mathematics Also the “teacher” may be a high school teacher, or x eallege insizetor, ar anyone interested in the weckinique of tesching mathematic, The ‘author looks at the situation eometimes from the poi of view of the student and sometimes from that of the teacher (the latter cate is preponderant in the fist pai). Yet most of the time (especially in the third part) the point of view is that of a person who is neither teacher for scudent but anxious to tolve the problem belore him. How To Solve It PART I, IN THE CLASSROOM PURPOSE, 1, Helping the stadent. One of the most important tasks of the teacher isto help his students, ‘This cask ie ‘not quite easy; it demands ime, practice, devotion, and sound principe. “The student should acquire as much experience of independent wark as possible. But if he it lft alone with his problem without any help or with insuficient help, bbe may make no progres at all. Ifthe teacher helps 100 much, nothing is left to the student. The teacher shoud help, but not too much avd not 106 Hil, 40 that the student shall have a reavonable share ofthe work If the student ix not able to do much, the teacher should Teave hitn at last some Musion of independent work. In order to do 40, the teacher should help the seadent discreetly, unobiristvey. ‘The best is, however, to help the student naturally. ‘The teacher should pu: himself in the student's place, be should see the student's eas, fe should try to understand what is going on in the student's mind, and ask a qu tion or indieate a step that could have occurred to the student himel 2% Questions, recommendations, mental operations. i t0 help the student effectively but unobtrusively nd naturally, the teacher i Jed to atk the same questions and to indieate the sime steps again and again. Thus, in ‘countless problems, we have to ask the question: What ‘i In the Classroom 1 the unknown? We may sary the words, and ask the sine thing in many difexene ways: What jp required? What do you went t0 find? What are you supposed to feck? The aim of hese questions i to focus the students Steention upon the unknown. Sometimes, we obtain the fame cifect more naturally with a suggestion: Look at the unknown? Question and suggestion aim at che same fffect; they tend to provoke the same mental oper ‘seemed to the author tha: it might be worth while © collect and to group questions and suggestions which are typically helpful in discussing problems wieh students “The Sst we study contains questions and suggestions of this sort, carefully choten and arranged: they are equally tseful tothe problenssolver who works by himelt. Ie che teaser i sufieienly acquainted wit the Hist and can se, ‘itn the suggestion, the action suggested, Ie may real ine thatthe list enamneraes,Indiecty. mental operations iypicaly useful for the solution of problems. These ‘Operations are listed im the order im which chey ate most ely to occur. ‘3 Generality fs an important characteristic of the qucations and suggestions contained in our ht. Take the Guestions: What 1 the unkown? What are the data? Wher us the condition? Tlhese questions are generally applicable, we can ask them vith good effect dealing ‘with all sorts of problems. Theit use ix not restricted to ny subjeceamatter. Our problem may be algebraic oF igeametcie mathematical or nenmathematical, theoretical br practical a serio problem or a mete puzzle: ft makes tno iiference, the questions make sense and might help tas to solve the problem, "There is a restriction, in fact, but it has nothing ¢© do with the subjectmatter, Cerlaio questions and cages Tiont ofthe list are applicable co “problems to find” only 5: Tescher and Student. tmitation and Practice 3 01 to "problems to prov." we have a problem ofthe Tatter Rind we must use different questions; sce vom ests 4 Common sense, The guctions and suggestions of cur list are general, but, ence for their generality, they ze acuta simple, obvioun, and proced fiom plain common sen. Take the suggestions Look at the um Anon! And ty to think of familar problem haute the soma or similar unknown. "This sogesion adios jou to do what you would do anyhow, without any Eve, you were secously concerned with your prob tem. Are you hungry? You wish to aban food and you think of familie ways of cbtining food. Have you a problem of geomesrie constriction? You wish 1 eon. Sue tangle and you think of familiar wae of ene structing 4 ingle. Have you » problem of any Kind? You wish to find a erain ‘unknown, and you think of femiar ways of finding such an unknown, or some sim ler unknown. Ifyou do #0 you folow exactly the sg gestion we quoted rom our a And you are onthe right Wack to; the sggetion i good a, sugges o ou 4 procedure whichis ery equentl sce ‘llth questions ed miggeton of or lit ate atu simple, cbsins jun plain common sense: bot they wate Plain common sene general terms. They sages 4 Eertaincordet which comes naturally fo any person who is seriously concerned with his problem and has some comiman ens Bu the peion who behaves the righ way ually does ot cae fo exptes his bebvioe tn leat words and: pssiy, he cnt exprs Ses] ou Ut is twexpres inva 5 ‘Teacher and student. Imiation an practice. There ane a which the teacher may hv view en AMtessing to his students » question ova suggestion of the lst: ir, to help te stant tn salve the problem 7 In the Classroom at hand. Second, to develop the student's ability v0 that the my solve future problems by hint ‘Eapetience shows thatthe questions and suggestions of four Tat, appropriately sed, very frequently help the Cradene ‘They have two common characteristic, comraon Senne acd generality As they proceed from plain common feove they very often come naturally; Uhey eould have Secured to the student himself. As they are general, they help unobtrusively they jost indicate « general direction dnd Teave plenty forthe student todo. “put the two aims we mentioned before are closely com ‘nected: if he student succeeds in solving the problem at hhand he adds a litle to his ability to solve problems Then, we should not forget that our questions are gear erst applicable in many eases. Ifthe same question fs repeatedly helpful, che student will scarcely fil to notice rind he wil be induced to ak the question by bimselt in asia situation, Asking the question repeatedly, he nay succeed once in eliciting the right idea. By such a cee, he discovers the right way of using the question, td then he has realy assimilated it "The student nay absorb a few questions of our Hist so well that he js nally able to pat (0 himself che right {question inthe right moment and so perform the corre ponding mental operation naturally and_ vigorously. ‘Such a student has cervainly derived the grestest possible prof from our list, What ean the teacher do io order € ‘brain this bet possible resule? ‘Solving. problems is a practical stil ike, let ws «ay, swimming. We acquire any practical skill by Wd practice, ‘Trying to eka, you imitate what otber people do with their bands and fet co keep their heads Thowe water, até, finally, you learn to swim by prac: Teing sutimming. Trying to rolve problems. you have tO ‘Gorerve and to imitate what other people do when sol 6 Four Phase : ing problems and, nally, you lear to do problema fig hem ule ee "The teacher who wits to develop ble stdent ails we do probe mat fil ome terest for robes tei minds nd ive them plenty of opportunity for ination and prac Ifthe teacher wie to develop in his sudente the mental operstions whic coespond to che quesaons and suggests of our the pus tere Ghesons and sggetons to the sudnis alten 3 be an do vo aturaly. Moremer, when the tcacher sles {probiem bnfore the cas, he should dramatise hi ideas 2 fide and be should porto nul! the me questions Shih ne se when helping the aden, Thanks to ich eidance the widen will eventually dover the ight She of these queions and suggestions, and doing so he equiv smething that more important an the Thowedge of any parila mathemati [ac MAIN DIVISIONS, MAIN QUESTIONS 6 Four pes Tying fd hea, we my xe peey ange str ola iw omy dng wethe blon, hve tosh cu pon agin nd tea ur coca of te roa ike be ttc’ eye oe evr mi a fl lea ne he al em pres pi tien ne we ve snc saad solution. a In are to group cone he qin ard sep fesen fa ae Sl tng poe Eerie weave otra the pres we i'w ec wht po we ae tx how he ate ems comet ho te un Tom's tke ote dr, ner saa ce titel emake on Threw you oe 6 In the Clessroom plan. Fourth, we Took back at the completed solutions Wwe review and discuss it ‘Each of Gheve phases has its importance. ¢ may ap- pen that a student hits upon an exceptionally bright Flea and jomping all preparations blusts out wich the olution. Sul hacky ideas, of course, are most desirable, ‘put something very undesirable and veforwimate may resul if the student Jeaves out any of the four phases ‘without having a good idea. The worst may happen if the student embarks upon computations of consiric wos without having understood the problem. It is generally wees 10 cary ovt details without having seen fhe main connection, of having made x sort of plan. ‘Many mistakes can be avoided if, carrying out his plan, the student checks each sep. Some of the best eects may be lose if the student fails to reexarane and to reconsider the completed solution 7. Understanding the problem. It is foolish to answer « question that you do not understand. 1s sad 10 wotk for an end that you do pot desce. Such foolish and sad things often happen, in and out of scoot, bu dhe reacher Should ty to prevent them from happening in his clas ‘The wwuden. shouki understand the problem. Bt he should not only understand it, he should also desire foluion. If the student i lacking in understanding ot ie Snterest it is not always bis fault; the problem should be ‘well chosen, not too difielt and not too easy, natural TIN interesting, and some time should be allowed for ‘natural and interesting presentation. “Frnt ofall, the verbal statement of the probe must bbe understood, The teacher can check this, up t0 3 «et Textent he asks the student t9 repeat the statement, ind the stedent should be able to sate the problen Ruently, The student should also be able to point out the priadpal parts of the problem, the unknown, the 8. Example ; Ata, the condition Henc, Ue teacher can seldom allord to inthe questions What i he snknownt het ane the dtc What i the eondtion? “The sadent should conser the principal pare ofthe problem atenivey, repetedly and from sao side reve age comes nth the poole he show row grt and point oot On the takrown an the dats, Ii ik newer t give eames to the ober he ‘ould introduce uate notaton; devoting tome sien, ton 1o the appropriate choice of sigs, he ged to consider the objects for which the sgn have to be chon ‘There is anower quesion which may be wal in tis Preparsory nage provided that we do-not expect 2 Active smewer bat juts provional anwer, 2 gure: Lit pouible to sts the condition? {athe exposition of Pat It [ps] “Underianding the probe aude itm sages “Geng se ausimed™ and "Working for better. uadersaning 1. Example, Let us lustte some of the pots ex Plaind inthe foregoing scion. We tae the felling Sinple problem: Fite dingnal ofa nectngatar part ipo which the tre ty nde hg order o dics thie problem profitably, the stents smut he familar wih the there at Pythagoras, and With sone of applications in plane geometry, Bu hey tay have very lite sjtematie Knowle in goon txry. The teacher may rey het pon te sven phe amy with sal elem 3 tacher cat make the problem interesting making tconcce The clasfoom ya Tectangol path ltepped whose dimernons could be mente, an can be enimated, the student hove © find, to “measure inteerty" the diagonal ofthe Sagmoe, "Te teacher pein oot ee len she wtih and te hight tthe 8 In the Clasr00r8 lasroom, indicates the diagonal with a gesture, and See Hi gure, drawn om the blackbose, by tlesing repeatedly othe soon. repre og Derween he tacher and tbe stuens nay tare as follows Syphat is the wnkown™ Me lenge of te agonal of parallelepiped.” “What ove te data” sot eng the wid, aa the height fe paral piped” ivoducesuileble notation. Which leer should Je pote the unkown “Rehien Jeers would you choo for the length the witha the Beght” “asboe” Shar the conto, inking 8 3802 CISL hisgont of the parallelepiped of WBC 0B, anc ane the nga ce with a the eight” wav vouonable problem? T mea, 1 the contin suffien to determine the wikncon®™ Heer ee tne ov 6, we krow the parallel piped tthe paraeleiped is determined, the diagooal ieetermined Devng a plan. We havea plan when we brow, oF eae e outine, whic exeaatons, computa teow ot CGauructons we have to perform in order fo Teta unknowns ‘The way fom understanding te oot concfing a pla maybe Jong a torn PRE pe maim achieve sm the oon of probe Tose omeatve the isa of plan. ‘Tis iden may sree gradually, Or afer apparently sawoecesal tat re of heaton, may ocurstidenly in 8 ran re nighe en.” The bet thatthe eacer can do pee dent isto procure for him, by snaberasve 9. Devising « Plan 2 Ielp a bright de. ‘The question and suggestions we ae Folie tedacn end oprovle dha er Tiare tobe abl fo see sont postion, ce teacher shoul unk of hi own eee aH il tole ces noting prbion oe ee steve lite Knowle of tesubjete and iopasibie tiv inet on Genta be opt experince and forme acid knowledge Mes rnessbering b nt enough for goo ies, bu we Gaonot hae tay pod tie wibout nalling sane portal, mater alone ne oot enogh fr cor Ffucting a house But we cnnec cutod a Boe wi tr alcng the necoarymatras The: mara es rage ene Tar tems of tar lormeryseqered mathemati Towldge, as former soe problems ot formes evel tnt i en spate at ost wi the ucts Dr jo Brow a relate eet 7 a let "Thc fit dha here ae wall are sally oo many prob- lens whic are ome elated ot Peta prem, tha hve sone olin common wit How cn we Choos the oe or he em whic are vey fl Thee io mgpenon tha pate or inget oma one eon tou poine Look othe wnlnon nd tte tink of Fier roto hag mo immo we cee in eli Torey ned role wc cet red to fur prevent problees we ae iy sly de ch sw teee by ening Te ere problem need to Yours and toed before Could you sei? "Te negoingqueon,welf understand eis comidced vey ten bcp to sac the righ tan of ido but they cannct help alway they anno wore we In the Clessoom magic. If hey do not work, we must look around for some biber appropriate polneof contact, and explore the vati- ‘un aspects of our problem; We have fo vary, to transform, to modify the problem. Could you restate the problem? Some of the questions of our list hine specie means to vary the problem, 28 generalization, specialization, use of {rnslogy, dropping a part ofthe condition, and so on; the Geils are important but we eanot go into them now. Variation of the problem may lead to some appropriate owls problem: If you cannot solve the proposed problem try to solve fist some related problem. ‘Trying «0 apply various known problems or theorems, considering various modifications, experimenting. with ‘rious wuailiary problems, we may stray so far fom our ‘original problem that we are in danger of losing it alto. eiher. Yet there is a good question that may bring us back to it: Did you wae all the date? Did you wse the tahole condition? 10, Example. We return to the example considered in section 8 AS we left it, che students just succeeded in Tnderseanding the problem and showed some mild inter tat in i. They could now have some ideas of their own, ome initiatives If the teacher, having watched sharply, ‘Cannot detect any sign of such initiative he has 9 resume Carefully his dialogue with the students, He must be pro pared (o repeat with some modification the questions Dinich the stedents do not answer. He must be prepared {fo meet often with the disconcerting silence of the fudents (which will be indicated by dot «++ 3) “Do you hnow a related problem?” ‘Leak at the unknown! Do you know « problem hav- ing the same unknown?” Swell, what i the unknown?” so, Example " “The diagonal ofa parallelepiped" "Do you know any prablem with the seme unknown?” "No. We have not had any problem yet about the agonal of a parallelepiped.” “Do you know any problem with a similar unknown? ou see, the diagonal ea amen, de segment of & traight line. Did you never ave # problem wow own wa the ei of line™ Ot courte ave saved such problems Frese, tofinda se ofa righ angle” “Gott Hee te probiem related lo youre ond tole before: Could you se” “You were lacky enough to remember 2 problem which is related to your present one and which you solved before. Would you like to use it? Could you introduce tome auxiliary element in order to make ifs use posible?” “Lock here, the problem you remembered is about a twimngle, Have you any triangle in your igure?" Let us hope that the list hint was explicit enough 10 provoke the idea of the solution which is to introduce 4 right triangle, {empbasized in Fig. a) of which the a In the Classroom rol diagonal isthe hypotenuse. Yer the teacher TRIuM be prepared forthe cine tat even thi ashy iid hist ioslicent co wake the torpor of he 8 hs and so he should be prepared 10 tse 2 whole grmutofmore and more explic ins Rola you ike to havea range nthe fie” “At sgt of eiange would you ike to have in che figure?” you eansot find yet the diagonal: ut you sid shat you cold nd he sde ofa tangle. Now, what wil you ie oud you Gnd dhe diagonal, iff were side of 8 crangle? ‘hen, evemualy, with more orl lp he students ote in toduelng the deci ailiary element, the gir clang euposzed in Fig the teacher should rence melt hat the wcidents sce sufiiently far ‘Thead before encouraging them to go into ctl cael Think hae i ws a goo den to draw tha vino ‘You have now a tangle ba have ou the unkwown?™ “Eye unkown the Bypotenuse ofthe trenle: We can eleatte ty the theorem of Pythagorn” "ou ca if bth leg are oven but ae they? “One legis genet se. And the oer 1 think, # not dita eG. Yeu the ote leg ithe hypotenuse of tomther ght range” very ght Nom Tac that you have plan.” TN cazeing oat the plan, To devie 2 plan, to cote ele the en the soloion i not esp Te ake 20 mh Sleds Torment nequired Knowledge, good ental ene Stcemation upon the purpox, and one mare TRREY oon Teck. To carry out dhe plan fs meh easier: hat we need aint patience tpn ives. a general eating; we have to convince Example ie ourselves that the detils fit into the ovine, and so we Ihave to examine the details one ater the other, patiently, sill everything is perfectly clear, and no obscure cornet remmalns in which an error could ie hidden TF the student hav really conceived a plan, the teacher hhes now a selaively peaceful time. The main danger is that the stadent forgets his plan. This may eaily happen if the student received hi pla from outside, and ac- ‘cepted it on the authority of the teacher; but if he worked for ic himself, even with some help, and conceived the final idea with satisfaction, he will not lo this idea ‘easly. Yet dhe teacher must insist thatthe student should check each step. ‘We may convince ourselves ofthe correctnes of 2 step fn our reasoning either “intuitively” or “formally.” We may concentrate upon the point in question till we see itso clearly and distinely thar we have no doubt that the sep fs correct; or we may derive che point in ques tom xccording to formal sles. (The difference between “inaight™ and "fgems? proof” is clear enough in many important eases; we may Ieave further discussion 10 philosophers) “The main poin: is thatthe student should be honestly convinced of the correcincts of each step, In certain cases, the teacher may emphasize the dfforence between “see ing” and “proving”: Car you see cleorly that the step is correct? Bua can you also prove that the step is correct? 12, Example. Let us resume our work at ibe point where we let Cat the end of section 10, The student, a last, las got the idea of the solution. He sees the right triangle of which the unknown » is the hypotenuse and the given height ¢ is one of the legs the other Jeg is the diagonal of a Tace. The student must, posibly, be urged to introduce suitable notation. He should choose to de rote that other leg, the diagomal ofthe face whose sides 4 In the Classroom are a and b, Thus, he may see more clearly the ides of the olution which is to introduce an auxiliary problem Sehote unknown is >, Finally, working at one right tt Tngle after the other, he may obtain. (Gee Fig. ») nas yoete xo Vetere TS Tae SE sa wae 0H oe 13, Looking Back date their knowledge and develop their ability to solve problems. A good teicher should understand and impress fm his students the view that no problem whatever is com pletely exiausted There remains slways something to do: with susicient study and penetration, we could improve any solution, and, in any case, we can abways improve our tunderstinding of te solution "The student has now carved through his plan. He has ‘written down the solution, checking exch step. Thus, he should have good reasons to believe that his solution i torrec. Nevertheles, rors are always posible, especially if the argument i long and involved. Hence, verifications are desirable. Especially, if there is some rapid and in itive procedure ta tes. etber the resul or the argument, it should not be overlooked. Gan you check the resuli? Can you check the argument? In order ta convince outelves of the presence or of the ‘quality of an object, we like to see and to touch it. And 5 we prefer perception through two different senses, 10 we preter conviction by to different procts: Gam you de: ve the remlt diferentiy? We prefer, of course,» short and intwtive argument to along and heavy one: Can you fevit eta glanced ‘One of the frst and foremoet duties of the teacher is not to give hs studencs the impresion that mathematical problems have little connection wich each other, and no Connection at al with anything else. We have a natural ‘Opportunity to investigate the connections of « problem ‘when looking back at ie solution. The students will find looking back at the solution really interesting if they Ihave made an hones effor:, and have the consciousness fof having dove well. Then they are eager to see what else they could accomplish with chat elfore, and how they ‘could do equally well another time. The texcher should encourage the students to imagine cases in which they 6 In the Classroom could utilize again the procedure used, or apply the re- ult obtained, Can yow tse the remlt, or the nicthod, Jor dome other problem? TA. Example, In section 12, the students finally ob- tained he solution: HE the ehrce edges of a rectangular pirallelogram, ised from the same comer, are a,b, 6 the diagonal is IPOS Gan you check the result? The teacher cannot expect 2 good answer to this question from inexperienced sta Gents. The students, however, should acquire fairly early the experience that problems "in le«ers” have a great advantage over purely numerical problems; if the prob- Jem is given "im letters its result is accesible to several {esis to which a problem “in umber isnot susceprible a all, Our example, although fairly simple, is suficient to show this. The weacher can ask several questions about the result which the students may readily answer with ‘r¥ex"; bat an answer "No" would show a serious lw in the result pid you use all the deta? Do all the data a,b, € appear in your formula for the diagonal?” ‘Length, widch, and height play the same role in our question; our problem i symmetric with respect 2 Bethe expression you obtained for the diagonal sym trie in a, 6, ef Does it remain unchanged when #, © are interchanged” ‘Our problem is 4 problem of solid geometry: to find the diagonal of a parallelepiped with given dimensions 4, b,c. Our problem is analogous to a problem of plane feometry: to Gnd the diagonal of a rectangle with given ‘Simensions a, bs the result of our ‘sotid” problem anal ‘ogous to the result of the plane’ problem” TE the height ¢ decreases, and nally vanishes, tbe 24. Example oy parallelepiped becomes a parallelogram. I you pute ~ 0 flyer ome >You bn te ce aria or agosl ofthe rectangular parlors "ft ight eicreney he agonal Ines. Does your formula tow this? "iLalt cree mesures a, 5, ofthe parallelepiped Sn crease in the me proportion, the diagnal alo increta in Se sae opr yon fmol yo tite axa bye fora be respetively, the expresion of the diagonal, owing co tht subttaon, sot sso be tulip by voto" “if, b, care mearired in et. your formula gies the diagonal card inet ov: bu yon change a et fs Int Ince the Soma should remain cored. I at Phe tq ea gunn Thee quenions have several god ee. Fn, an n- seTigent steers cannot be beng pred by the face thatthe formula pac to many tte was canine telore thatthe formule ore Beare e derived it {Sty But now het more conned ani gain fSnidence comes rom a deren sone, fa 0 3 torr of experimen evidence” Then thanks to the foregoing queons, the dais of the formula aelre ew tigiftance, and ave inked up with varow fee "The tora far tertove a better chance of being fe smembered, the knowlege ofthe audet i comaidsed Finally the quesocs ean been ranted to si lar probes. After some expeiene with sinilar prob Jems an nelignt sade may pers the undering fener dente of al selena, sviaton of toe Sei symmetry, anor. Hb got taka the Babe of Girecing hie avention to sach port, hai solve roles ay detely prot. 8 In the Classroom ‘Can you check the argument? To recheck the argument sep by step may be necesary in dificult and important tates. Unualy, itis enough te pick out "touehy" points for rechecking, In our case, It may be advisable to discuss retrompectively the question which was less advisable 10 discust ab the solution was not yet attained: Can you prove that the triangle with sides x,y, ¢ is a right tr langle? (See the end of section 12) ran you ute the sevult or the method for some other problem? Widh a litte encouragement, and after one or ‘iso examples, the students easly find applications whiel, ‘consist enenially in giving some concrete interpretation to the abstract mathematical elements of the problem. ‘The teacher hime used such a conczeteinterpresation fs he took the room in which the discussion takes place forthe parallelepiped of the problem. A dull student may Propose, as application, to ealeulate the diagonal of the ‘Cafeteria instead of the diagonal of the classroom. Ifthe stadents do not volunteer more imaginative remarks, the teacher himgelf may put a slighty diferent problem, for instance: "Being given the length, the width, and the height of a rectangular parallelepiped, ind the distance ofthe center irom one ofthe corner.” ‘The students may use the result of the problem they just solved, observing that the distance required is one half of the diagonal they just calealated. Or they may vse the method, introducing suitable right triangles (the later alternative ix less obvious and somewhat mare clumsy inthe present case). ‘Aiter this application, the teacher may discuss the con- figuration of the four diagonals of the parallelepiped, And the six pyramids of which the six faces are che buses, the center the common vertex, and the semidiagonels the lateral edges When the geometric imagination of the ‘students i saficiently enlivened, the teacher should come 15. Various Approaches 19 back to his question: Can you use the result, or the method, for some other problem? Now there isa better ‘chance that the students may find some more interesting concrete interpretation, for instance, the following: "In the center of the fat reciangular top ofa building which is 21 yards long and 16 yards wide, a flagpole isco be erected, 8 yards high. To support the pole, we noed four equal cables. The cables should start from the same point, # yards under the top of the pols, and end at the four corners ofthe top of the building. Hlow long is each cable?” ‘The students may use the method of the problem they solved in detail incoducing a right eriangle in a vertical plane, and anther one in x horizontal plane. Or they ‘may use the result, imagining a rectangulst paralcle piped of which the diagonal, x, is one of the four cables and the edges are ars ba8 ems. ‘By Straightforward application of the formula, x = 1445 For more examples, see CAN You WE THE RESULT? 15. Various approaches. Let us stil yetan, for a while, the problem we considered in the foregoing vections 8, 10, 12, 14. The main work, the discovery of the plan, wat escribed in section 10. Let us observe that the teacher ‘could have proceeded differently. Starting from the same pint asm section 10, he could have followed a somewhat ‘iferent line, asking the following questions: “Do you know any related problem?” “Do you know an analogous problem?” "You seth prope problem ia problem of wld geometry. Could you think of a skupler analogous prob Tem of plane geometry?” ze 2 "You see, the proposed problem is about a figure in space, it in concerned with the diagonal of a zectangular 2 In the Classroom parallelepiped. What might be an analogous problem About a figuze in the plane? It should be concerned with the dixgonal—of—a rectangular ‘Parallelogram.” ‘The students, even if they are very slow and indiffer- ‘ent, and were not able to guess anything bei ‘obliged finally co contribute at lest a minute part of the dea. Besides, if che studenss are so slow, the teacher should not take up the present problem about the para: Jelepiped without having discussed before, in order 10 prepare the students, the analogous problem about the parallelogram. Then, ke can go on now as follows: “Here is a problem related to yours end solued before. an you use i" “Should you introduce some auxiliary element in order tomate its use posible?” Eventually, she teacher may succeed in suggesting to the students the desirable idea. It consists in conceiving the diagonal of the given parallelepiped as che diagonal of a suitable parallelogram vehich rust be introduced Into the gure (as intersection ofthe parallelepiped with 2 plane passing through two opposite edges). The idea is festentially the same as before (section 10) but the ap- proach is diferent. In section 10, the contact with she Available knowledge of the students was established ‘through the unknown; « formerly solved problem was recollected because its unknown was the same 25 that of ‘the proposed problem. in the present section analogy provides the contact with the idea of the solution. 16. The teacher's method of questioning chown in the foregoing sections 8, 10, 12, 14, 15 is ewentially thi Begin with a general question oF suggestion of our list, 1, Hf necessary, come down gradually to more specific and concrete questions or suggestions Hl you reach one which elicis s rexpomse in the student's mind. If you 16, The Teacher's Method of Questioning a1 have to help the student exploit his iden, start agen, if pouible, from a general question or suggestion contained and return again to some more special one if ‘and 10 on, Of course, our lst is just a fast List of this kind; it seems to be suficient for the majority of simple cats, but ‘there is no doubs ha it eould be perfected I is impor tant, however, that the suggestions from which we start should be simple, natural, ad genera, and that chit list should be shor The suggestions must be simple and natural because ‘otherwise they cannot be smobirsve, ‘The suggestions must be general, applicable not only to the present problem but to problems of all sorts, if they are to help develop the ability ofthe student and not justa special technique. ‘The list must be short in order that the questions may be often repeated, unartificially, and under varying ci- cumstances; thusy there is a chance that they will be ‘eventually aimilated by the student and will contribute (o the development ofa mental habit, Tt i neccesary to come down gradually to specific ug: ‘gestions, in order that the student may have as great @ share ofthe wark 3s posible. This method of questioning is not a rigid one; for tunately 50, because, In those matters, any rigid, mechan ‘al, pedantical procedure is necossarily bad. Our method. admits 2 certain elasticity and variation, it admits various approaches (section 18), it can be and should be so applied that questions asked by the teacher could have occurred to the student hirset fa reader wishes to try the method here proposed in Is dass he should, of coursc, proceed with cuution. He should study carefully the example introduced in section 8, and the following examples in sections 18,19, 20. He a In the Classroom should prepare carefully the examples which he intends todincuss, considering also various approaches. Te should ‘art with a few tials and find out gradually how he can manage the method, how the students take it, and how rch time i takes. 1, Good questions and bad questions. If the method ‘of questioning formulated in the foregoing section is well Tunderstood it helpe to jodge, by comparison, the quality of certain siggestions which may be offered with the in- tention of helping the students ‘Let us go back to the sicuation as it presented iself at the beginning of section 10 when the question was asked: Do you know a related problem? Inszead ofthis, with the best intention to help the students, the question may be fered: Could you apply the theorem of Pythagoras? “The intention may be the best, but the questi is about ‘the worst. We must realize in what situation it wa of fered; then we shall see that there is a long sequence of objections against that sort of “help.” (a) If the student is near to the solution, he may un: derstand the suggestion implied by the question; but if Ihe isnot, he quite possibly will not see at all the point at which the question is diving. Thus the question fails to help where help ie most needed. Ii the suggestion is understood, it gives the whole secret away, very litle remains for te stoden: to do {@)_‘The suggestion ie of too special a nature. Even if the student an make vse of it in solving the present problem, nothing is lerned for future problems. ‘The MG) Been il he understands the suggestion, the student can seaeely understand how the teacher came to the idea of puting sues » question, And how could he, the st dent, find such a question by himselD Tt appears as an unnatural surprise, as @ rabbit pulled out of a hat; itis really not instructive. 18, 4 Problem of Construction 5 [None of these objections can te rated against the prot cedure described in section 10, or against that in see tion 15. MORE EXAMPLES 18. A problem of construction. inscribe a square i a given triangle, Two vertices of the square showtd be on the base of the triangle, the two other vertices of the quare on the tao other sides of the triangle, one om each, “What isthe unknown?” “A square.” “What ere the date?” “A wiangle is given, nothing else.” “What is the condition?” “The four corners of the square should be on the per- imeter of the triangle, wo corners on the base, one or rer on each ofthe other :wo sides.” “sit possible to satisty the condition?” *Tedink wen not so sure" "You do fot scem to find the problem too ess. 1] you ‘cannot salve the proposed problem, try to salve first some ened problem. Could you sii 2 att of th con “What do you mean by 2 part of the condition?” "You see, the condition is concerned with all the ver tees of the square. How many vertices are ther plates "A part ofthe condition would be concerned with less than four vertices. Keep only a part of the condition, Arop the other port. What part of the condition is e1sy tosatity “It is easy to draw a square with two vertices on the perimeter ofthe tiangle-ot even one with three vertices fon the perimeter!” “Draw a igure!” a In the Classroom The student draws Fig “You hept any fir! of the coniton, ad you dropped the other part. How far i the wnkncum now fleterminer”™ “Phe square is not deteroined if it has only three vertices on the perimeter of the wiangle” “Good! Draw a igre” The eudent deans Fig. 6. “The square as you tid is not determined by the part of the condition you kept. How can it vary? meer of “Thee corners of your square are on the peti th rangle Dut the fourth comer i not et here where iu should he. Your square, as you sii, i undetermined, 19. A Problem to Prove 5 i can vary, the same is true ofits fourth corner. How con ievarge “Try experimentally, if you wish. Draw more squares, with three corners on the perimeter in the sime way a» fhe evo squares already inthe figure. Draw smnall squares and large squares. What seems to be the locus ef the fourth corner? How cen it vary? The teacher brought the student very near to the {dea of the solution. If che student is able to puss that the locus of the fourth corner is sieaight line, he has ori 19. A problem ta prove. Two angles are in different planes but each side of one is parallel to the correspond ing side of the other, and has ao the same divection, Prove that such angler are equal What we have to prove is a fundamental theorem of solid geometry. The problem may be proposed to ste dents who are samiliar with plane geometry and a0 quainted with those few facts of solid geometry which Prepare the present theorem in Eucld’s Elemente (he theorem thar we have stated and are going to prove is the Pyopestion 10 of Book XT of Fucld) "Not only ques. tions and suggestions quoted from our list are printed in Ualios but also others which eottespond to thezn as “problems to prove” correspond to “problems to find.” (The comespondence ix worked out systematically in PrODLEMS 70 Mi, PaoBLEMS TO PROVE 5 6) What isthe hypotheseh” “Two angles are in diferent planes. Each side of one 4s parallel to the corresponding side ofthe otic, and has to the seme direction, “What isthe conciusion?” “The angles are equal “Draw a jigure. Introduce suitable notation.” 6 In the Classroom ‘The sendent draws the lines of Fig. 4 and chooses, helped more ot les bythe teacher, the leurs as in Fig. 4 “What tthe hypotesst Say i, please, using your nota- “4, B, © are notin the same plane as 4’, B’, @. And AB|A'B', AG | A'C’. Also AD has the same ditection as ‘A'B' and AG the same as 4°C." “What isthe conclusion?” “ZBAC ~ LB aC: “Look atthe conclusion! And try to think of a familiar theorem having the some or «similar conclusion.” “IE wo triangles are congruent, the corresponding ngles are equal” "Very good! Now here is a theorem related to yours and proved before. Could you use it?” “Tthink so but I do not se yet quite bow." "Should you introduce come auxiliary element in order to make its use possible?” “well. the theorem which you quoted so well is about 19. A Problem to Prove "7 triangles, about a pair of congraeat any triangles in your figure “No, But I could introduce some. Let me join B to C, and BY to G Then there are two ulangles, ABC, BARC! “Well done. But what are these triangles good for?" “To prove the conclusion, BAG = 784°C." “Good! IF you wish 10 prove this, what kind of ti angles do you need?" jangles. Have you ‘Congruent tangles Yes, of course, I may choose 2, ,8',6's0 that AB AB AC~ ate" “Very good! Now, what do you wish to prove?” “T wish to prove that the triangles are congruent, A ABC = 4 ABC ALT could prove this, he eonctusion ¢BAC = 2 B'a’C Would follow immediately.” “Fine! You have a new aim, you aim at a new concla sion. Look at the conclusion? And try to think of 8 In the Clasroom familiar theorem having the seme or a similar conch "Two triangles are congruent if-if the three sides of the one are equal respectively 10 the three sides of the ree ‘Well done. You could have chosen a worte one. New here is theorem related t0 yous and proved before Could you use i?” “cold we iif Tknew that BC = BC’ “That is right! Thus, what is your ait? “To prove that BC = BC” “tp to think ofa familar theorem having the same ov ‘similar conclusion.” “Yes, 1 know a theorem finishing“ Tines ave equal” But ie Goes not fic in.” "Should you introduce some auxiliary element in order tomate its use possible?” “Yea see, how could you prove BG ~ Re" when there {sno connection in the igure between BC and B/C" ‘Did you use the hypothesis? What isthe hypothesis?” “We suppose that AB I 4°B", AC 1 4’C’. Yes, of course, msust use that Did you wie the whole hypothesis? You ssy that AB i “APB Ts that all that you know about these lines? ‘Not Alig also eqal to A°B', by construction. They are parallel and equal to each other. And so are AC and Two parallel lines of equal length it is an interesting, configuration. Haze you seen it before” OF course! Yes! Parallelogramt Let me join 4 to 4’, Bt9B,a0d CC." The idea ip not 40 bad. How many paraliclograms have you now in your figure Wo. No, dee. No, two. { mean, there are two of 20, A Rate Problem which you can prove immediatly that they are parak Ielograins, There ie thire hich seems to bea parle sgram; | hope Team prove that i sone, And ‘thea the [root will be finished” We could have gathered frm his foes 7 2s foregoing snewers hat the sudent i tellgent. Bu fers las remark of his, hee ino doube ‘This sent fable fo guess mathematical esl and to dixinguish clery between prof and gues. He knows aio that guenes can be more o les plausible, Really, he aid prot something trom his mathematics clase, he has some real experience in solving probemy, he can concive and exploit good ice 28. A race problem: Weler is lowing ino a conicat essel atthe rte The see! has the thape of aright Circular cone wth horizontal ase, the vertex pointing downards; the radius ofthe bates the ate of Ihe 6.6 cone Find th ne which the sac issn when th eth ofthc tri Fly bn eet valu of te untnan ssppting tht a= fob fr FB I pr mines end y= fe i ee he eb ae ppc Yo know the simple rues cl iterensntion tnd tc notin ef “rte ange “Whatarthe a “0 In the Classroom The radius of the base of the cone a ~ 4 ft, the al tude of the cone & = 3 {, the rate at whieh the water is owing into the vewsel r—2.et, fL per minute, and the depth of the water a a certain moinent, y~ 3." EComect. The siatement of the problem seems to sug. _gest that you should distegard, provisionally, the mumeri- fal values, work withthe letters, expres the unknown in terms of eb 9 and only finally, after having obsained the expresion of the unknown in letters, substitute the ‘numerical valuex, T would follow this suggestion. Now, that ie the unknown?” “The rate at which the surface is ising when the depth cof the water is.” “rwhat is that? Could you say it in other te “The race at which the depth of the water ceasing.” “What is tha? Could you restate it stil diferentiy7” “The rate of change of the depth of the water “That is right, dhe rate of change of 7. But what isthe mte of change? Go back tothe definition.” “The derivative isthe rato of change of a function.” “Correct Nov, is y a function? As we said before, we isregard the munerical value of y. Can you imagine that y ehanges?” “ves, 7, the depth of the water, increases asthe time goes by “Thhus, 7 is a funetion of what?” OF the time & “Good. Introduce suitable notation, How would you vite the “ate of change of 9" ip mathematical symbol?” dn a Good. Thus, this is your unknown, You have t0 ex pres itin terms of , bY. By the way, one ofthese data fsa ‘rate’ Which one?” 20. A Rate Problem o “ris the rate at which water is flowing into the verse.” ‘What is that? Could you say iti other terms” “ris the rate of change ofthe volume of the water in the vee.” “What it that? Could you ret How would you write it io sutable not eB Whats “The volume of the water in the vesel atthe time t" it stilt dierentty? hart “ood, Tho you hve tenes in ems 5 © tho wily doi I] Jou cannot solve the prepored problem try to solve first some related provtem. It you do not see yet the con- nection beeween don betneen and the data, ry to bring in some simpler connection that could serve asa stepping stone.” “Do you not see that there ae other connections? For wiance, ate y and independent of each other?" ‘No. When y increases, ¥ must increae 100.” “Thus there is 2 connection. What isthe connection?” “Well, Fis the volume of a cone of which the altitude iy. But Ido not know yet the radius of the bas "You may consider it, nevertheless. Ca it something, says.” ey 3 “Correct. Now, what about 22 Ts it independent of 92” ‘a, When the depth of the water, 9, increases the radius of the free surface, x, increases too." “Ths, there is @ connection. What isthe connection?” a In the Classroom of coun, similar triangle miyeest 18 not miss ‘one more connection, you see. T woul prong from it Do not forget, you wished o know the Eonnecuon between Vand 5.” L have s2¥ 5 atte vam : epping stone, does it Very gods This ook Tike 2 epping sod een shold not forge Your Eon, What the unkown nye," well ov hve fod a oonecondetcen 2, a vet other quantities. And here you have o find other quantities. What to do?” Diderentiare! Of courset wee. ane a Frere itis" | : “Pinel And what about the numerical values 7 EXOXI dy . 9 a PART II, HOW TO.SOLVE IT ~ A DIALOGUE 9.347-735-A Getting Acquainted Where should I start? Start froma the statement of the problem. What con I do? Visualize the problem a 2 whole 28 clearly and as vividly as you can. Do not concern your: self with detais for the moment. What can I goin by doing so? You should understand the problem, familiarize yourself with it, imprest i pu ‘post on your mind. The attention bestowed on the probs Jem may also stimulate your memory and prepare for the recollection of relevant points Working for Beuer Understanding Where should I tart? Stare again from the statement of the problem. Start when this statement is 40 clear 10 you and so well impressed on your mind that you may Tose sight of it for a while without fear of losing it ether. What com I do? Isolate the principal problem. The hypothesis and the conciision are’ the brincipal parts of a “problem to prove; the unknown, the data, and the conditions are the principal parts of 4 “problem to find." Go through the principal parts of your problem, consider them one by one, consider thea in turn, consider them in various combinations, relating each detnil to other detnils and each to the whole of the problem, 3 st A Dialogue ould prepare and What cam I gain by doing so? You should preps carify details which are likely t play a role afterwards Monting for the Helpful Tea sree soul Loe Set ram he conden of ake ined pao our poem, Sar eh «Prete dare ciainty atranged and cely cm- Pra fe a your previous werk, and When YoUt Semon scomsrevpcsive srg tan aap oie your pele om various ange ona wits your formerly segue nme : zi Mee your problem fom vaxow ie. Enphsst Ce ea cmv dierent dea examine Se ealnrapetely ot in adres ys combine Sse cimaenayy sppsouh hem om diferent Sty nme sew es nh Se Me cepearonattiewhole Se wih our trey acqited koowled® soy te wha bed yrs la in tye eogaieeonveting lami Wha ea To peel somching wea 5 wt YU eget ald 1 perce A belt iden, perhaps de orn Ae foo yo at anes he Wy 10 EY scan an idee be Delf shows you we whole of eee Tear ae way ge You Eo Bee how yeu com proceed Hews are move sept, You ae Ivey He yo have 29 i Ove ‘can I do with an incomplete idea? You should i ehooks ientageo ou shou omit comer oaks ail You shold eri A Dialogue i. far it leads you, ond reconsider the situation. The situa sion hus changed, thanks 9 your helpful idea. Consier the new situation from various sides and seek contacts with your formerly acquired knowledge, What can I gain by doing s0 again? You may be bucky and have another idea, Perhaps your next ides will lead you to the solution right away. Perhaps you need # few more helpful ideas after the next. Pethaps you will be Jed astray by some of your ideas. Nevertheles you should, be grateful forall new ideas, also for the leser one, also for the hazy ones, abo for the supplementary ideas add ing sonte precision to a hazy one, of attempting the ear. rection of a les fortunate one. Even if you do not have any appreciable new ideas for @ while you should be {grateful if your conception ofthe problem becomes more ‘complete oF more coherent, more hemogencous or beiter balanced. ‘Camrying Out the Plan Where should 1 start? Start from the lucky ides that Jed you to the solution. Sart when you feel sure of our rasp of the main conncetion and you feel confident that You can supply the minor details that may be wanting What con I do? Make your grayp quite secure, Carry through in detail all the algebraic or geometric opera: tions which you have recognized previously at feasible. Convince yourself of the correctness of each step hy for: ral reasoning, or by incuicive insight, or both ways if you fan. Tl your problem i very complex you may distin Buish “great” steps and “small” stops, each great sep being composed of several small ones. Check fitst ene reat steps, and get down to the smaller ones afterwards What can I gain by doing so? A presentation of the solution each step of which is correct beyond dou 8 A Dialogue Looking Back Where shoul tee From ie sosion complete and comet inch deta nar dat Consider de slain from vaio et ntact your formers seq Toowlaee ue des of te woaton and ty 30 make eer as yu cn suey more exer ats sa jut and try to make Sem shart 10 $= sa toon wen ince Tey to reodiy 0 ei oe sro ger perso he san, 0 Heyden mae sing ro smery acqused anowiedge se tsa) 3 ie SGutnne te mend that Te Ou 10 the Fee tose pital ey to make se of for saat. on Sree seal and one wS eto our problems. ean pa dng? You may fn nr A Fe oe any dover new andere Hae ene it you get sno the habit of sying ee Ie athcng your altons in cis wa, 300 will aac Efomatge wll tered andrei 0 8 se i develop Yor abiy of ahing poses PART II. SHORT DICTIONARY OF HEURISTIC Analogy is 2 sort of similarity. Similar objects agree veith ead other in some respect, analogous objects agree In certain relatonr of thes respective parts 1. A recangular parallelogram Is snalogous to @ rec tangular parallelepiped, Tn fact, the rlaons Beveeen the ides of the parallelogram are similar to these be tween the faces ofthe pralelepipe Each side ofthe paralelogram is parallel to just one other ide, and is perpendieuar yo the remaining sides. Each face ofthe poralelepiped is parallel to just one other face, and is perpendicular to the remaining face Tet agree tall se a “bounding element of the parallelogram and face a "bounding element” of the parallelepiped. ‘Then, we may contrac. the two fore fing sttements inte one that applic equally to bosh fare: ach bounding clement is paralle! to just one other hounding clement and is perpendicular tothe Femainiag boonaing elements ‘Thus, we have exprened certin relations which are ‘amnion othe two sates of abject we compared, sides of the rectangle and faces of the recangulaparalele Piped. The analogy of thee spews contain ths come Inunityof relations, te Aualogy pervades all our chinklog, our everyday speech and our evial conclusions as well as atic ways of expresion and the highest aciemuSe achieve tents Analogy i used on very diferent levels, People a * Analogy ten we vague, ambigvoos, incomplete, or incompletely Ghesbet anstogien but analogy 7may reach the level of ethceatea precision, All sorts of analogy may play & Mie inthe dtcovery of the solution and so we should tot neglect any ort ve may consider ourselves lucky when, tying t0 soles problem, we suceed in discovering a simpler hugo problen. In section 18, out original problem sre earned sith Ge diagonal of rectangular para Tesi the consideration of simpler sralogos probs wea ncemed wit te diagonal of &fectangle Hl v0 the suena the orignal problem. We are going t© cnr one more ese of the same sort We have to save the following problem ‘Hind the eter of gravity of &@ Romogeneovs tetre- heron ‘Weanout knowledge ofthe integral cles, aed with inte Rnontedge of physic, his problem i not easy at stars lout stenife problem in the days of ‘Mists on Galen. Thu, iw wis to solve with Aika ftininary knowledge as posible, we should iho toda for a stmpler anslogove probler. Te coe wepang probit inthe plane oecus ere natural lad a center of grenty of @ Komogeneous triangle Now. me have wo questfons instead of one: Bat (0 unsere may be eset to aera Jost one question areidad tat the two questons are intclligently com mma, nr Laying asi, forthe moment, ov orga problem cofceatng he teahedron, we concene spon he ‘Sthpir seatogous problem concerning he sang, To sre his problem we have to Know something about sae te prvi, The flowing principle is plausible fond prevents elf narra Mesto of masse 8 conse of fats, eck of whch Avalos ony 99 deine an in ins also the center of gravity of the whole system ‘This pince yt al fr we ned tet of wg Fie imps ta ae oa ae eae wage er tn fae he ge Tn fe Slt sng cong a ss sini sart pagan pes os seo te tg he si ah ints ine ae Stay af nd any palin) se sy ep ad ei ee ine ing sec € see ae aidpoln a ole @cetig he c A M 8 «227 ioe peng trou he meta CA fe angle cnn he et of ray fal pre ay whe cite se eg Thos ne te edt he casion thatthe center of gravity ofthe whole teh Sng les on the sane median, Yet it muse le on Oe other two medians jatar wel mast be the common pelt meron of ter nen beable 1 verily ow by pure geometry, i pendenly of sty mechanic, sumption tht the tee ‘eins etn the sae poine © Analogy After dhe case ofthe triangle, the ease of the tet hedion is fairly easy, We have now solved problem soalogoun t9 obr proposed problem snd, having solved it, we have a model to follow. {Jn rolviog the analogous problem which we use now a samuel, we conceived the triangle ABC as consisting of bere parallel to one ofits sides, AB. Nov, we conceive tbe tetehedron ABCD as consisting of fibers parale) to one of is edges, AB, "The midpoints of the Abers which constitute the angle le al on the some straight line, a median of che “langle joining the midpoint M of the side AB to the ‘pporite vertex €. The midpoints ofthe bers which com slate the tetrahedron Ke all in the same plane, joining the midpoine Mf of the edge AB to the opposite edge CD {Ges Fig 8); we may cll this plane MCD a median plane of the tetrahedron. D tn the case of the triangle, we had three medians tke Me, each of which has 19 contain the center of gravity fie triangle: Therefore, thee three medians must meet none point which is precy the center of gravity. I= Analogy - Sone eee Pale eeepc a pede eee came oe as teeated sae ona eins Soe ae co eee Mee ate ida ee ae ane ns oe peckabed egrets ee ee as on Se Reni eee aes ea seoe ene Perey ie Anatogy eich pase Oough A and abo ofthe § median plaee Tee" Bs ana alo of the g thiough C. Connecting thas fac suablye we may prove tat de © median ites hve a earmon poe (The 3 median planes tg through the side of BC determine & coe inna nes of iurection which ree the ender eine. Now, by. what we Dave jus proved Ghrough en ine of tsetection one more mesh plane oth under and wer 6 ye we simpler aalr 7, publ, eomering the eng, to solve » el TOURS cretion es the ew cas as dierent ‘Ra important respect. Unde we wed the method of the imple analogous problem bose slvion we fm purty pote Unter & weaned the res of the ee Matigs peoiea, and we did not ce Bow Teicha nd beeu obtain Sometimes, we may Be fbi nba he method and he eat oe spe ee ious protem. Ever out foreguing example shows Tae Eiger abe conideraions under 5 and 6 8 {Brent pant of he soon of the sme problem ‘Sur cetmples ypc. in solving a proposed problem. swe cam often Te olton of wpe naogoss ce may bese tow awed o se repo Ofcourse, wore dul ave, complicaions ot lsc are nt yet abown by out example TRLGING, Icio appen the the wlution ofthe ano. cape bien cannot be immediately ved for our Og Fee ean Then, i zay be wort while 1 recosidr wre Buon, vay and to snot il afer having RP iows forme ofthe solution, we find eventually Sho aar ean be encended our ooeinal problem. 4. ie desable to free he elo at me secu of resale wit woe degre of plu ey Sloable forcast oon bas on analy Anatony ia Thus, we may Know Gat the center of gravity of a loctgencou langle coincides wit the centr a graisy ois three vers (chat of thes rnc pny wit qual mca, placa inthe veces of the angle), Buowing thi we tay cinjecore that the cone of fravty of a homogeneous tctrason coils with the Conn of gavotte vertins ‘Tas eonjete an“inerence by analog” Know tin the angle and the etraecion ve ae in any repeats, we conjectre thatthe ae lle in one sore Fees e wrod be foot op the plasty fide conjeoures at creas, but ie would be lt Tooth, or even more fol to digas plasble nectar Thlerence by analogy appeats be the nest common Lind of conctson, andes pony the mest ental Ld. I yields more dr lew plamible conjecores which say or ay not be confine by experience and cet featcing ‘The chonit, experimenting ov animal trder to foresee the infoence of his drugy on human, Gras cochsons by analogy. Hot to dda sal oy T new, His pt do had tobe taken the vteroay, ar be inquired: “Who is the veterinary?” "The animal doctor: “Which animal isthe animal doctor?” ‘9. An analogical conclusion from many parallel cases in stronger than one from fewer cases. Yet quality is ill ‘more important here than quantity. Clearcut analogies weigh more heavily than vague similarities, systematically arranged Instances count for more than random collec: tons af eases. In the foregoing (under &) we put forward a canjec- ture about the center of gravity of the eetrahedron. This conjecture was supported by snalogy: the case of the “ amawygy tetrahedron is analogous to that of the triangle. We may ‘rengthen the conjecture by examining one more snalo- four case, the case of a homogeneous rod (that iy a Staight linesegment of uniform deasiy). “The analogy beeween segment wiangle thas many aspects. A segment is contained in a straight Tine, a rangle ina plane a tetrahedron in space, Straight Tinesegments are the simplest onedimensional bounded figures, wrangles the simplest polygon, tetrahedrons the simplest polyhedron. "Tne segment bas 2 zerodimensional bounding ele cs interior is one-dimensional "The wriangle bas § rerodimensional and 3 oneclimen sons) Bourlling elements (3 vertices, g sides) and its interior is t9o-dimensions "The setrahedron bas 4 zeredimensional, 6 one-dimmen- sional, apd 4 twodimensional bounding elements (4 Tertices, edges 4 faces), and its interior i dhree-dimen: sonal. "These numbers can be sisembled into a table, ‘The suc: we ealumns contain the numbers for the zero one foro. and thee dimensional elements, the succesive rows the mumbers Zor the segment, rangle, and tetrahedron: tetrahedron 330 aeat Very ite fatarty withthe powers of binomial i Met cognse in ese suber section of Pascal’ reer ne found «remarkable replat i segment, triangle ané tetra. erat a fave experienced that the objets we cone pd a omy conte, "inferences by analogy.” a BM owing, py havea cevtin weight with Analogy ms “The center of gravity of hmogeneous rod eoinciin sen the center ot prvi oft» ea poins The omer Seay of homogenous tingle nice ih the Centr of gravity a te 5 vetes. Should we nov vet that he center of gravity of homogeneous eesnon Coincds with she eee of gray a he vere? ‘Agin, the camer of prety of a homoqeacos roe vies the dinance swe i ed point te proper Som 414 Thecemer of avy of langle dives the disence beeen ny veren andthe mifpoine othe opposite sige nthe proponon 9 howls we net ue ee thatthe cee of avy of a homopeneout etre Sevan vies she tance tone any seven ad the ce of he pp ine en Teappean extemely wail thatthe conjure geved by seve quesion shold be wrong tat sud ust reglaity shouldbe spo "The fcting that tarmonious simple order cant be ser uies the discoverer both in the mathesnatal aod inthe she sence, end exproned by the Latin saying. simplex gon sr (mpaciy che seal ot rt. (The preceiog sug an extension to dmersons Xe appear unity that what fe inthe fs tee Gimeno for n=, should cee t be wae lor bigher values of mT conjetnae tan "inference by indesign lastest tion snarl bed fn analgy. See maeerion ano Matters move We ish dhe presen section by considering briety the mos portance in which analogy stn the Precio of hema es (@ Trosyems of mathematical obec say Sand, axe connected that certain tlatons teen the 08 Jess of S are governed bythe sine las tose betcen the abject 6 Auailary Blements “This kind of analogy between $ and S" is exemplified boy what we have discussed under 1; take aS the sides of 2 rectangle, as 5 the faces ofa rectangular paralilepiped. {AD ‘There is » one-one correspondence between the objects of the two systems S and 8 preserving certain Felations, Thet is, f such a selation hokls bevwcen the objects of one system, jon olds beeween the corvesponding objects of te other system. Such a Connection between two systems is 2 very preci sort of ftalogy, it i called Horaorphism (or holohedral iso morphism) lil) There is a onemany correspondence between he objeets ofthe two systems S and S” preserving certain {elations. Such a connection (which is important ia var ‘us branches of advanced mathematical study, especially in the Theory of Groups, and need not be discussed here in detail) is called merokedial isomorphism (or homo ttorphism; homoiomerphismn would be, perhaps, a bette term). Merohedral isomorphism may be considered as another very recive sort of analogs] Auxiliary elements, There is much more in our con ception of the problecy atthe end of our work than was fiie a we started working (PROCESS AND ACITEYEWSNT, 1) As our work progresses, we add new elements to those Suiginally considered. An element that we introduce in the bope that it will further the solution is called an scuxilery element ‘There are vasious kinds of auxiliary elements, Sov ing» geometric problem, we may introduce new Tines Iino our figure, euxiliary lines. Solving an algebraic prob- Team, we may introduce am ausiliary unknown (AUXILIARY Tomes, 1). An auifiary theorem is a theorem whose proof we undertake inthe hope of promoting the solution ‘our original problem. Auxiliary Elements 7 There are variows reasons for introducing aus elements We ae glad when we have suceeded in rc Teeting a problem related to ours and solved before. Is probable that we can use sucha problem bur we do not Foow yet how fo seit. For instance, the problea which ve ate tying to sale isa geometric problem, and the Felsted probleat which we have solved betare and have how soceceded in reolecting is © problem about tr gles. Yet sere is bo triangle in out Egues in oder to snake any use ofthe problem recollected we must ave a triangle; therefore, we have to introduce one, by adding Stable auxiliary lines to ovr Rare, In general, having recollected formerly solved related prottem and wis ing to te it for our present one, we must often 2 Should we introduce sme cusiliary clement in order f0 snake ite sue possibi? (The example in section 10 is ‘pica Going back to definitions, we bave another opportw ny to lneroduce auxiliany cements For instance, explt- Catlng the inion of circle we should not only mention center and ts radius, but we should aso Introduce these geometic clement into out Sigur. With out introducing them, we could not make any concrete tie of the definition; stating the definition without drawing someting is mere lipservice “Trying to we hyown teuls and going back to defn iors are among the best reasons for introducing Savy el = nts; but they are not the only onei. We may ‘dd auxiliary elements tothe conception of our problem in order to make it fller, more suggestive, more familiar although we scarcely know yet explicily how we shal be able to use the elements added. We may just fee! that it ie a “bright idea” to conceive the problem that way ‘with auch and such elements added. ‘We may have this or that reason for introducing an 8 Auaiiary Elements ‘auxilary element, but we should have some reason. We Should not introduce availiary elements wantonly. S Example. Construct 2 triangle, being given one angles the altitude drawn from the vertex of the given and the perimeter ofthe triangle. A vag P “We introduce suitable notation. Lata denote the given angle bse given alte drawn fom the vertex of fed p the given perimeter. We drow a figure in whic Meeully place a and h, Have we wed all the data? Nov ‘ur figure does not contain the given length p, equal 0 Auxiliary Blements 49 the perimeter of the triangle. Therefore we must intro duce p But bow? ‘We may attempt to insreduce p in various ways. The attempts exhibited in Figs 9, 10 appear clomsy. 1! wet fo mike clear to oureelver why they appear to unsstie. faciory, we may perceive that itis for lack of symmetry. In fact, the iiangle has dvee unknown sides a,b, We cal a, as usual the side opposite to 4; we know that abbteep. Now, the sides b and ¢ play the same role; they are inter: changeable; our problein is symmetric with respect to & and c. But # and ¢ do not play the same role in our Figures g. 10; placing the length p we treated & and c iflerently, the figures g and x0 spoil the natural sym retry of the problem with respect to & and c, We should place p so that it has the same relation to b a 0 « This consideration may be helpfal in suggesting to place the length p as in Fig. 14. We ad to the side « of the triangle the segment GE of length on one side and the segment BD of the length c on the otber side so hat appears in Fig, 1 asthe line ED of length beatenp, Af we have some little experience in saving problems of fonsisuetion, we shall not fail to introduce into dhe ° Auxiliary Problem figure, slong with ED, the auxiliary lines AD and AE, een ot which is the Base of an isosceles sriangle. In ft, Seis not unreasonable to introduce elements into the problem which are particulary simple and familiar, a4 Teosceles wiangle. ‘We have been quite lucky ia introducing our auxiliary tines, Exarsining the new gute we may discover that EAD has a sme relation to the given angle «In fact ‘Ne find using the isseeles triangles 4 ABD and 4 ACE that £DAE = $+ go" After this remark, iis nator to try the construction of A DAE. Trying this construction, Wu introduce am auxiliary problem which is much esse than the original problem. ne ‘Teachers and authors of textbooks should not forget thde the intelligent student and THE INTELLIGENT READER re not satisfied by verifying chat the steps of & reasoning, dre correct but also want to know the motive and the purpose of the various steps. The introduction of a Pnitiary element is x conspicuous step. If a tricky atuuiiary line appears abruptly in the gure, without. mand solves the problem surprisingly, intelli ent sundenus and readers are Gisppointed: they feel shat Shey are cheated. Mathematics is interesting in vo far as it eecupies our easoning and inventive powers But thete is nothing to learn aboot reasoning and invention ifthe motive and purpose of the most conspicuous step remain Tncomprehensble, ‘To make such steps comprehensible bby suitable remarks (es in the foregoing. under 2) or by refuly chaven questions and suggestions (as in sections TO, 18, 19,20) takes alot of time and effort; but it may be worth while. blem is a problem which we consider, fs opn sake, but becaase we hope that its con Asilary Problem » sideration may bel derstion may help us to wl annter problem, ox orignal robles. The original problem ithe end we Sr to tain, the aailany problem a means by which eatin nd ict tres to scape tough the windowane trie the ame again and agai, ae dvs no ry he nen indo wich open a through which it cme nto Shee Aah ate or ald ee ot amore Fnteligenty, Human superiority consists in ging Bound an abwacle tat cane be overcome dtely ie Sevising a stable cuir problem wen te origi problem appears insoluble. To devise an aay prob- Eine an imporiact operidon ofthe mind. To Pave 8 cleat new probiem sabievient to another problem tocanerive dane asm end what ena Yo anber tnd, ira tefied achievement of the avligene Tes an important tauk to lear” (orto teach) bow to handle bury problem intel. 1. Bxtnpl ind saying the cquation soigttsino, If we obperve that 24 = (e8)? we may see the advan tage of inteducing “ems * yo We ob o win now a new problem: Find 9, satisfying the equation aus yo iptsone. ‘The new problem isan auxiliary problem; we intend to ig eas of sing ou igi! probe, The sekoonn of ot sein probly. apn 2. Example. Find the diagonal ofa rectangular pars lepiped being given thelr of fron same nar" On am Sawn se Auailiary Problem “trying to wie this poblen (ation 8) we may be ved byt analogy, (etlon 19} 10 another probe Tika the dngnat of» senoglr parallelogram bing Ret the engulf ewe side drawn fom the me “Ts ew problem ian ausilay problem: we consider i: becuse We hope to dsve some postr the aig tem fom comer. rtf The prot tht we derive from the consider axbin fon airy prob may be of varios Kinds ney dee eu tte onlay problem. Te £2 SEL cing found by solving the qundraic equ GOP f tae yis equal to 4 oF to we dnfer that Boul Zab gand derive hence all posible vanes of Sfp diner ca we may se te method of se nly ten Thin example» the suary probes 2 Preble of plane geometry, ti analogous fo, but sim eget oigal problem which i a problem of spiny. Ts esonable co introduce anni) seer ts Kind inthe ope that it wil be irae oe itl ive us opportunity to Lamiliaite out tee ath crtan meted operons, or tol, which SES oe aterards for ur nga problen. Imex we he cote af the svilary problem i rather a ening cely we And that we cn we Doth BOUL Si andi vesle Gee secon 15, and i» rou tse me me a7) ws We tke away from the original problem the vie ne ent thatwe deve Lote oy prob Te Pr ioveigaton of she airy probe "Eethcand ore we devoted eo ma Be kon. Ther toe Soul exerci our judgment in. choosing 20 ser eeven, We may have tarious good Taos Pie ‘The nusny problem may peat He (ore dh the orginal problem, or Ht MAY APPEAL Ausiliary Problem io inset of may have sor of eed appa Shuts ner and were arene postin we eee ale ane ae STlpprcctto whic sem ob erated Secale ie Sheep oe main of aad payed oie ein depends on the doer a Pde anny pen: Unlapy, exe bai Jane mated of ocoverag ate sua prt. tens tere inte eed of dcveig the Deere ee rpee reed bine age eee renee % Equivelent problems. Two problems are equivalent st the iodonofceh vs he ston of oe _ espe reer reer Mulan problem aceqlene pe eer ee Bee eq angle eq: in any equlngsar wing, ech ange gi in ay engin wiols ec age "These evo tear ae not iene They eneain ideas Soe oe cnc ih equ of te Sidhe coer win equity af he angola are Bees tren tie toe oer Tree he priest prove Ah equa ie prolem to roe ee ge ising wos mang prbion he prot lem o prove B. The theoem Bi lide ener to prove then kc hat 8 mene nova We my foe a pare a raagieee Pe eeinmeener pecan ay ey oa Ausitiary Problem the theorem B, concerned only with angles is more “Homogeneous” than the theorem A which is concerned ‘with both angles and sides. “The pasiage from the original problem to the aus: itiary problem is called convertible reduction, oF fatcrol redvetion, or equivalent reduction if these two problems, the original and the auxiliary are equivalent Thus, the reduction of A to B (ee above) is convertible and so is che reduction in example 1. Convertible reduc tions ae, in a eriain respect, more important and more Gesieable than other ways 10 introduce ausiliary probs Tem, but auriliary problems which are not equivalent to the original problem may aso be very wseluls tke ‘example 2 y Chains of equivalent ouxilory problems are tre quent in mathematical ressoning. We ate required £0 tiipe a problem A; we cannot see the solution, but we Tay God tae A is equivalent another problem Bs Considering B we may run into a third problem C equiy- ‘Stent to B. Proceeding in the same way, we reduce C10 ‘Drand vo on, until we come upon a last problem L. whose Sclution fs known oF immediate. Each problem being ‘Squivalent to the preceding, the last problem L mst be tivalent to our original problem. A. Thus we are able fe infer the solution ofthe original problem. A sro dhe problem I which we attained as te last link ina chain Df anxiliary problems Chains of problems of this kind were noticed by the ‘Greek mathematicians & we may sce from an important Dosage of parvos. For an illustration, let us reconsider Bar cxample 1. Let ws call (A) the condision imposed ‘upon the unknown x: ” ato ight soo (One way of solving the problem is to tansform che pro- Ausiiary Problen Po ‘posed condition into another condition which we shall all @ oy Gs)? 2) ast yo. ‘Observe that the conditions (A) and (B) are different ‘They ate only slightly diferent if you wish to say £0, they are certainly equivalent as you may easly convince yoursll, bur they are detinitely not identical. The pas {age rom (A) +0 (B) Ss not only correct but has a clear ‘cut prpore, obvinos to anybody who is familia with tbe folution of quadratic equations. Working Same dizection we transform the condition (B) another condition (C) © (ast) 2 a (ax?) 15 4 16985 Proceeding in the same way, we obtain ) (axe agt= a5, wo axt ag © © ay a gor—g.or nor —& Each reduction that we made was convertible. This, the Jas condition (B2) is equivalent to the fist conditios (A) 60 that g, —3, 2, —2 are all posible solutions of our original equation. Tr the foregoings we derived from an original condi tion (A) a sequence of conditions (B), (C), (). each of which was equivalent to the foregoing. “This pojint deserves the greatest care, Equivalent «i lve satisfied by the same object. Therefor, if we pass from a proposed condition to 2 new condition equivalent o Auxiliary Problem to it, we have the same rolutions, But if we pass from a propored condition to a narrower one, we lose solution’ pied we pass to a wider one we admit improper, adver tidous solutions which have nothing to do with the pro- posed problem. If, in x.serie of successive reductions, we Pw 108 narrower and then again to 2 wider condition eeinay lose wack ofthe orginal problem completely. In Syaer to avoid this danger, we saust check carefully the frature of each newly introduced condition: Is € eq lent co the original condition? This question is still nore important when we do not deal with 2 siagle equ tom a ere but with systema of equations, oF when the ‘Condition # not expres by equations as, for instance, {im problems of geometric construction. {Compare raves, especially comments 2,8; 4,8. The deseiption on p. 143, lines 4-81, i» unnecessarily re Siriteds it describes a chain of "problems 10 find,” exch of which has a different anknown. The example con Shdered here has jus che opposite speciality: all problems ‘Of the chain have the same unknown and differ only {9 fhe form of the condition. OF course, no such restriction isnecesary.) 8. Unilateral reduction. We have two problems, A and 3B, boi unsolved, If we could solve A we could hence derive the fell solution of B. But not conversely, if we Could aulve B. we would obtain, posibly, some informa tiom about A, but we would not know Bow to derive the full solution of A from that of B. In such a case, more is ehieved by the talution of A than by the solution of B. Let us ell A the more ambitious, and B the less ambi tous ofthe two problems. IL, fom a proposed problem, we pass either to # more ambitious of to a Hess ambitions auxiliary problem we aT the step a unilateral reduction, There are wo Kinds Sf unilateral reduction, and both ate, in some way oF Bernard Boluano Pa cies, more ety than a bilateral or convert ‘duction a date Out example x shows a wnsteral reduction to a lew om problem. In fat, we coud salve the orga problem, concerned with a parallepiped whowe lng, “and abe reprciney, we cd ore Onto te auxiliary problem puting ¢ = and obtaining 2'yaallogems with lenge and wii . For anode Sete our eon ew inn problem se sacituranion, 9,45 Tho examples show Shah wie ea ae les a oe snlay problem a8 stepng stone combining the slucon of the atin problem with some appre Prise supplementary femazk to abla the wlio of the oil problem Unilateral recon toa more about problem aa ali be race Sucre, yh ere thon of the fx 19 the second problem comer In the more eon may Deore Boleano, Bernard (1781-1848). logician and mathema Lien, devoted an extensive part of his comprehensive presentation of logic, Wissenschafislehre, to the subject ‘of hewrisic (Wo. 5, pp. 295575). He writes about this pate of his work: I do not think at all that Tarn able to present here any procedure of investigation that was tot perceived long ago by al mea of talent and T do not promise at all that you can find bere anything quite new ofthis kind, But T shall ake pains to state in clear words the rules and ways of invesigation whieh are followed by all able men, who in most case are not even conscious of following them, Although T am free from the ilasion that I shall fully suecced eves in doing this, 1 sl hope 8 Bright Hea that the Lede that is presented here may please rome people and have some application afterwards ei ie, po iene “cig he ig” Sa cigs decrbing udder aban 0¥a" oa tna nema 6 The wos rg iden sam experience fara to every it gett describe ands it ay be intern ny a ery aggestive escrpion of as bee ey ven Dyan ality aso 8 Arial Seer i agree that contin aight deat seepage arse deties “gnc” 2 fl Ter Sse hit by goes upon the ese do cae an inappreaable ine. As for expe, ees cron aang with 27h man #92 certain Yo sey meal goes tht that person ng 2 Trane. Or ebsring tha de Brightside of the teem ry omar the nu, You may sul er aly bene te mon shies ye crea th Me aeape nt bad But rather ei nt a edd og gy of set abet ah ey ani dn mt very Dh Ta et ors ample however, qe imprente ve a ee agian vo en ORE ay es ould realize that a contemporary of Aristotle had aoe ae esa srs he wabed wan Che vo wat ae wee no arate, and bad 10 ob ee the moon ite planed waving OY Teenie were re wet light, He wat mck nig ie hrs te ay than the imodeen ly Ae SN arpy of the Fasc Stes (80 velit Gan You Check the Result? 89 dweller, and his natural intelligence was not dimmed by ‘undigested fragments of journalistic preentations of astronomical theories He si the fll moon aa Bat dis, similar to the dise of the son but much les bright. He ‘must have wondered a the incessant changes inthe shape and postion of the moon, He observed the moon o¢r3- slonally also at daytime, about sunrise or sunset, and found out "that the bright side of the moon is always toward the sun” which was in itself 2 respectable achieve- ‘ment, And nov he perecives thae the varying aspecss of the moon are like the various aspects of « ball which is iMluminated fom one side 19 that one al of itis shiny land the other half dark. He conceives the sun and the ‘moon not as flat dises but a8 round bodies, one giving and the other receiving the Wight. He understands the fexeentil connection, he reamanges his former concep tions instantly, “in am inappreciable time”: there js a sudden leap of the imagination, a bright idea, a fash of genius. ‘Can you check the rexule? Can you check the argue ent? A good answer to these questions strengthens our trust in che solution and contributes tothe solidity of our knowledge. 12, Numerical results of mathematical problems cam be tested by comparing them to observed numbers, or to 4 commontenie estimate of observable numbers. As probe lems arising trom practical needs or natural curiosity almose always aim at facts it could be expected chat such ‘comparisons with observable fact are scldom omitted. Yer every teacher knows (hat students achieve incredible things in this repeet. Some students are not disturbed avall when they find 16,150 ft. forthe length ofthe boat and 8 years, p months for the age of the captain who is, bby the way, Known to be a grandfather. Such neglect of 6 can You Check the Result? ‘the obvious daes not shave necesarily stupidity but rather Fnuilflerence coward artifical problems. saiproblem “in letter” are susceptible of more, and mage jtezeating, tests than “problems in nusbers™ (= Gon 14). For another example, Tet us consider, the num of a pyramid with square base. If the side of the lower base i a, the side of the upper base ®, and the altitude of che frustara hy we find for the volume abate 3 “We may test this resale by sepcuatrzaion. In facts if pace the frosturn becomes a prix and the formu Jak oth; and if ~ the fustum becomes a pyramid ad he Somoula yes! We may apply the 155M rseseon, In fac, the expres has as dmiession the Sane ots lent Agni, we sma test she formela by ci of he dats tfc, anyone of the poiive Tere, or knees he salve of the expression “Tost eis sort ean be apied no only othe at rel bur aot intermedi el They 360 eal PU ota hie preparing for the: ae vauaTon sea Seon nour to beable owe sch st Sr ed avaotage is genabing a “problem in Menard chang it oto a “problem fn ete: lev crnanatas108 “an yn chee the exgument? Chechng the a «nana pe, me should awd mee repetition. Fst iment jana apo become boring unnstueies ae ree alm, Second, where Westen on Sen ey ay omble gan the ciCOmEANCES ete ae before fe foe hat it necessary 6 £0 ae ang the whole argatentstep By ep we soa Gan You Derive the Result Diferentiy? 61 at Tee change the order ofthe wep, or Uni grouping feinotucrsone van Tn Be i Ic requires ls encruom ad is sore teresting to pik ote vn i eae cai Feist A quesion very well tn pking oot pots eee areeen ee ree seat pena? eis cen that our nonmathemtica Laowtedge can nel te ted emir on formal roots Tae move ct fart of our eeryy knowlege in continually tested and rergtened by our every experince Tess y ob- Svition sre more sematially conducted inthe at te iene. Such tee abe the form of catefal Sxpetments and meairenents ad ate emmbined wih sini eng ns ppl em Co wledge in tatheatics be bved on formal ros alone? : Thi is «philosophical quetion which we eanat de eter Ics eran ht jour Haowedge, oF my koov tige- or Yur student’ hnovledge in athemads i not Taal on formal ros alone theres ay slid know tige Stall it ht a broad experimental bas, and his aus is broadened: by each problem. Whose result i siccentuly eed Cam you derive dhe rel iferaty? When the sla sion tne we have Sly bane ng ad involve, we raturaly spect that there it ome clearer and Te Toundaboot soluon: Cam jou derive the rel dier ny Can you sei ef a fone? Yet even ite have Succeed io fining mst soloion we may sll be tected in Rnding another soon. We dre ( = Mince ourself te vaity ofa theoretic rele rate anivadon te reo nae ta Fv objec though sya dierent see, Having found 3 Gz Can You Derive the Result Diferenth? proof we wish od another poo as we wih ovo objet after having sen Perot ne beer than one “ie safe ing mt wo anchors” eee 2 Faemple: ind de area $of the ral sate dhe ean ot night Grelar cone, being given che iMeaeof the tower tse Khe ac of de upper Bs Tn the alitde “Phe problem can be solved by varus procedure Foe insane we may Know te fore for he lateral wee BMRA Gnu te eum gener by eating of frown a coe a sales cone, a i tral sarc the {irene of wo fall oni sofaes i emai 1 6 coer pain ean ff, Catying though side, Mr sbainfally he foraula 5-1 VEER Having ound ths reste m some way oh ater Kenge cduuladon, we may desre'2 Cewer and es 1erBSatout argument: Ga ow devee te rent ir run? Con yoseit ats ene? ‘Dairng eo se Inlely the whole rel, We say lepn ch yng to se the geomettic meaning of paves Tha wermay observe ht va sete length of the slant ight, (he slant height oe ithe nooparai sds of the foxes waperoi@ th, Shlving aboot the ine joining the midpoint of 3 Fale le, gnerotes te sts Se Fig. +8) Aa wre may dicuer hat oka RED isthe arithmetic aean of the perimeters of the ewo bases Can You Devive the Result Differently? — 63 of the frustum. Looking at the same part of the formula, ‘wemay be moved to write also in the forza Rg HRA =a that is the perimeter of the midsection of the frostum, (i¥e call hore mid section the intersection ofthe frustum wich a plane which is parallel bath tothe lower base and to the upper base of the frustum and biscets the alizude,) Having found new interpretations of various pacts, may $e now the whole formula in different light. We say read it tho Area ~ Perimeter of midaection X Slant height ‘We may recall here the rule for the taper Area = Middle line X Altitude (The midaledine is parallet to the two parallel sides of the trapezoid and bisects the altitude} Seeing intuitively the analogy of both statements, chat about the frustusn fand that about the trapezoid, we see the whole result about dhe frustum “almost ata glance.” That is, we feel 5 Can You Use the Reselt? that we are very neas now toa short and direct proot of the result found by along ealslation, The foregoing example i typical. Nox entirely sat fied with our derivation of che test, we wish to imyprove she to change it Therefore, we study the result trying (0 Understand it better, 10 ee some rew aspect of it We nay scowod fist in observing a new Sntepretacion of & ‘antain aall part of the result. Then, we may be lucky fnough to discover ome new mode of conceiving some cther past Tauining the various parts, one afer the other, and ving various ways of considering them, we may be Ted fealy to ee the whole regltin a dierent light, and our fnew conteption of the resale may suggest a Rew proot Temay be confesed that al this i more Tkely co hap pen to an experienced mathematician dealing with sme Pivanced prabem than to a beginner struggling widt ome elementary problem. The mathematician who has grea deal of knowledge is more exposed than the bei Bey tothe danger of mobilizing coo much knowledge +d framing an unnecesarily iavolved argument, But, & fompemation, the experienced mathematician és in & fetter position thae she beginner to appreciate the rein texpreation ofa stall part ofthe result and co proceed, vaeimulating such smalt advantages, co recasting wit: nately che whole result. Nevertheles, it can happen even in very elementary clases that the studente present an unnecesarily ¢0= plicated solution, Then, the teacher should show ther, Frente once oF twice, not only how co solve the problem rere shortly bur also bow t0 find, in che rewult tel, ileations ofa shorter sol ‘See also BADLCTIO AD ARSUREUM AND INDIRTCT PROOF. ‘Can you use the result? To ind dhe soktion of 3 prob teen by our own means is a discovery Hf the problem is Can You Use the Result? 6 sot difical, hedicovery is not so momentous, but it 2 discovery nevertheless” Having mace some iiseavery, homever molt, we should no lv Ingle wither there something more beind i, we sbould nota the Prstiilides opened up by the nr raul, we should oy {2 mw agun the procedure aed. Exploit your ses Gan you ws the tet, or the method, for some other problem? 1. Wee exiy imagine new problems if we ate sme hat familar wh the principal means of varying a Problem, a8 GENIAALIZATION, SPECALZATION, ANAL DecoMrostnG AND nEcoMINING. We start from a proposed problem, we derive irom ioe bythe tans We ust Zenioned, rom the problems we obtained we derive till other, an so on. ‘The proces unlimited in theory bn pace, we acim ctr very ar bene the problems that we obtain so are apt coe inseentbe ‘On ihe oller and we eo conse new: probleme Which we can ely solve sng the sation of problem Free eb tse ty new Pole tobe uninteresting "To find anew problem which i both inteenting ad accel i not fo not doen; we need eeperiene, cst, and food liek Yet we should not fil to lok sound for ‘ore good problems when we have ceeded in solving ne. Good problems and moshroors of erin Hinds ave something in common; they grow fn cuter. Hav ing found ones you should lock sround: there good chance tha there are some more quite ea. 1 We ate going to utate some of the forego pin by the same example tat we discussed in sections 8,10, 12 14, 15. Thus we start from the follow Problem: ied Foi te sce inendom den, revi ad eight) ofa rectangular paralclepped fd the diagonal we now the salu of thi problem, we can aly 65 (can You Use the Result solve any of the following problems (of which the frst fovo were almost stated in section 14). “Given the dhvee dimensions of a rectangular patallele: piped, find the radius ofthe circumscribed sphere. "The base ofa pyramid is 2 reciangle of which the cen- ter is the foot of the wtivude of the pyramid. Given the Sitivude of the pyramid and the ses offs base, find the lateral edges. ‘Given the rectangular coordinates. (xy. Fur #2)» (fay ‘yor) of to points im space, find the distance of these ‘We slve thee problesns emily because they are searcely diferent from the original problem whose solution we know. In each case, we add some new notion to our orig mal problem, as rcumscribed sphere, pyramid. recta ‘coordinates. These notions are easly added and ‘Rail eliminated, and, having got rid of chem, we fal ‘back upon our original problem, “The foregoing problems have a certain interest be- cause the notions that we introduced into the original problem are interesting, The last problem, chat about the Uixance of two points given by their coordinates, is even fan important problem because rectangular ooordinaics sre important. ‘Here is another problem which we ean easily if we know the solution of our ociginal probles the length, the breadth, and the diagonal of a 4b This however is Woe, fori is equlalent to abs i>e fe-ye>e ad hs inequality esaily Mok, utes = tat therecangiexained ra gute ‘ve nace not aed our problem et, bute have nade tne prgies just y facing sgtrey 0 rather hina gues Tt dmathematia enple. Ina certain crossword pelt ve ove to nda worl with even eters andthe Hive ee the allo agin, ack and fot.” hace noun? A woe What ave th data Ube eng of dhe mor is givens has vere eters i recondition esate nthe cue, T as someting too ith wally tel ery hay Thr Se have orexamin thetic As we do th vat pty eth oo enon: sain, back and ee" Could you sole prt of the problen? Here + ‘Mince to guts the begining af te woe. Stee the ‘pluton oo stongiy eophasized, the word, ite Teese igh are wih ver” Tis prey abovs rae etare wemped to elee i e shoot ale Sie ime The out erqited wou ook ss RES an you check the result? another word of the pt ale croses she one just considered im the fist Teter, we have an R cg star chat other ward. Ht may be a good iea 1 The Nain. ue Figures 105, to switch to that other word and check the R. IE we anceeed in verifying that Ror ify at east, we do not find any reason agaicst it we come back to cur original word. “We ask again: What is the condition? As we reexamine the clue, the very last part may catch our attention Tack and forth.” Could this imply thar the word ‘we seek can be read not only forward but backward? "This is a les obvious guess (yet there are such cass, see eoonsrosine AND RECOMEEINING 8) ‘At any rate, let ut face this gues; let us realize what it ‘means. The word would look 25 follows: RE---ER, Morcover, the dhind letter should be the same asthe ith; it is very likely a consonant and the fourth or middle ewer a vowel ‘The reader can now exily guess the word by hice. JE nothing elve helps, he can try all the vowels, one ater the other, forthe leter in the middle Figures reno only the objet of geometric poems bat tho am inportan help fora soc of prolen in hich there i ohn geomet at he onset, Th We Tere good reo to conse te sole of gues saving wei oe pabiem ie» probe of geometry, me have to omider gre Thi ore may bein oo gia to, or tay be uaced on paper Oe erin oon, itmight be dart ofage the Bre without de ing it bar Ine hve to amine vr deta one Atal fer he ober i deoale co draw gre Ik shee ave many deat, ee cannot imagine all of hea Snutanere but they meal togter othe pape, ‘deal pitted in our Snagit nay be forgets Bat the deal ted on paper rei dy when We 04 Figures come back to ie it reminds us of our previous remarks, it {avers some of the trouble we have in recolecting our previous consideration. We now consider more specially the use of figures in problems of geometric constriction ‘We strt she detailed consideration of such a problem by drawing a figure coneaining she unkown and the uta, all these elements being assembled as it is pre feribed by the condition of the problem. In order 10 tunderstand the problem distinetly, we bave co consider tech datum and each part of the condition separately: thea we reunite all pirts and consider the condition as whole tying to see sinnultaneously the various connec: tions required by the problem. We would scarcely beable to handle and separate and recombine all shese details ‘without a figure on paper. ‘On the other han, belore we have solved the problem. defnttively, it remains doubtial whether such figure tan be drawn at al, I it possible to satis the whole cov: dition imposed by the problem? We are not entiled to fay Yes belove me have obtained the definitive solution: nevertheless we begin with assuming a figure in which the unknown is comnected with the data as preseribed. by the condition. I scems that, drawing the fuze, we have rade an unsearranted assumption. ‘No, we have not. Not necessarily, We do not 2c incoe rectly when, examining ovr problem, we consider the ponibitity that there isan object chat satisfies the com- EEfdon izaposed upon the unknown and has, with all the lata che requited relatious, provided we do not confuse tere possibility with cerainty. A judge does not acti orvectly when, questioning dhe defendant, he considers the hypothesis that the defendant perpetrated the crime J question. proviced he does not commit himself to this hypothesis Boch the mathematician and the judge may ‘reamine a possibility without prejudice, postponing the:t Pigures ae Sicgment sl te exainstion pik’ some dete re sult. is = The method of waning the examination of problem of consracion by Eeaving ¢ sketch on ohh Mp Prey te condo td goes back tothe Grek rometr I ined by the sry somevtat eigetc Parse o Pappu dame what reputed tobe don Utreaty dove, Te tlloring recommendation some tat letra but eter’ raw s Iporhetal fue hick spores the conden of te prcoem ste it Mla gore nits + recommendation for problems of geometric consi bf act ther 0 ne to eee to Sy sch part kind o proc, We may exe the ‘eemendstion tal “peoblen fo fe” nating iin the following general orn: Examine the Iypoenal tution nic the conditn othe problem sup pried be aly sate Compare attic 4 Lets dca ow pio about he aca ra whee polos abou he actu dawing {Stall we draw the guise exacy or approx my tans tefont Winds of gue have their advange. Exact Agare ave i pine, he eae Tle game tact mertremnt in phys bn pty ext foresee important than exact mearremens De Ci th here omy mh meen Shy verted shun the awe of py, Toe beginner Rowers should contact many Rese exaely at Fe can in order to asuite a god experimental basi ad Ser may segs one ares a we ove alvencd er Jor he porpow of reuoaing, re fay drawn techand fgwret ae tava goed cough tn chey ave much move quity one Gf couse, the Bure ould ‘ot tok atvurd Tin sppoed tbe 105 Figures singh should not be wavy, and socal Gee howd otk ike poste Stiacctie igre can ocsonlly suggest» Se conthaon, but the danger i not grest 208 We es [™ eee enties from it by vious means, pecially Dy Shine the gare "Thee in danger ie concentrate Vp ha ection a ene ta the Pe det by no means the Das four conten fa WpPat conteaionsconsitate the reat bas. [Phi Mine's unrctvelyasate by cetin well how Beret which apis cleverly the itenonsl fc ae of te Sgr. 0) 1s orane thatthe cements are ase in he required lado, Fe unimportant in which Xe ae come ertoe, owe te ra se ond, Foren, to sate the Hes of ‘Sei jon wo ra tv aes «2, ha as eaing trom ar aban You sno! t= Siac with rar and compass, Teen, you cone Se ll, bur aerwie abitary B and, stng Gat you amore 9 whic ey Mb, Nour yur sould not set any wee spe cillnion Te difevent par o the gre sould ot care Sparen enone oot requited bythe problem. Ce REIE nt sccm to be equal ott be perpendics HIS. they ae no neesuy 0. Telongle shoul eS ae be boxes ov rightangled. wes no woe re Sy rele byte prablen. The angle having wort asda yar te one whic, in» peck ec sends be sine emore™ both rm the Sein thm the sgheanged shape You xe one at Se eames al OS 4 we eS ae a Figures i this, or a not very diferent triangle, if you with to con: idera “general” tiangie, QV) In order to emphasize the diferent roles of di ferent Lines, you may use heavy and light lines, contin ‘vous and dotted lines, or Fines in diferent colors. You ‘draw a line very lightly if you are not yet quite decided to we it as an auxiliary line. You may draw the given clements with red pencil, and use other colors to em plsize important parte, ata pair of similar triangles, ete (V)_ In order to illustrate solid geometry, shall we use three dimensional models, er drawings on paper andl Dlackboard? aa ‘Threesdimensional models ate desirable, but trouble- some to make and expensive to buy. Thus, wsvaly, we rust be satsed with drawings although it is not easy to make them impressive. Soie experimentation with seltmade cardbourd models fe very desirable for besin: ners. It is helptul to take abject of our everyday sur roondings as representations of geometrie notions. Thus, a box, a ule, oF the classroom may represent a vectangu Jar parallelepiped, 2 pencil, a circular ylinder, a lamp. sade, ce frustam of a right circular come, ete. 4 Figures traced on paper are cisy to produce, easy 0 recognize, easy to remember. Pare figures are especially familiar to us problems about plane figures especially accesible. We tay take advantage of this circumstance, Wwe may use our aptitude for handling figurcs in handling ongcometrical objecs if xe contrive © find 4 suitable ‘geometrical representation for. thote -nongeometrical objoct, fact, geometrical representations, graphs and die: rams of all vorts, are used in all seiences, not only in Physics, chemistry, and the natural sciences, Dut a0 in economics, and even in psychology. Using some suitable seometrical representation, we ty to express everything 108 in dhe language of figures to reduce all sorts of problems to problems of geometry. “Phos even i your ober i not a prob of gem eur, your may try 10 draw « figure. To find a lucid geo- metic representation for Your Hougeometrical problem ‘ould be sn important step toward the solution. ‘Generalization is passing from the consideration of one object co the consideration of & set containing that ob- ject or passing trom the consideration of a restricted set fo dat ol @ more comprehensive set containing the restricted one. 1. If by some chance, we come actos the sum 18+ 27+ 64 = 100 swe may observe chat it can be expressed in the curious form what ght goo Now, it is natural to ak ourselies: Does it often happen that asin of successive cubes a5 we aty ghee nt isa square? In asking this, we generalize. This generaliza- tion Js # lucky ones it leads from one observation to a remarkable general Lae, Many results were found by Tucky generalizations in raathematic, physic, and the natutal wiences, See INDUCTION AND MATHEMATICAL, R- ‘2. Generalization may be weful ia the solution of problems. Consider the following problem af solid geom- Cary: "A straight line and a regular octahedron are given Jn position. Find a plane that passes through the giver Tine and bisects the volume of the given octabedion. This problem may look dificule bu, in fact, very Kile Generalization 109 familiarity with the shape of the regular octahedion is uicient to suggest she followirg more general probles: "A stright line and a solid with a center of symmetry are given in position. Find a plane that panes throug the given line and bisecs the volume ofthe giver soli” ‘The plane roquited passes, of course, through the center ‘of symmeuy ofthe solid, nd is determined by Unis point and the given line. As the octabedron has a center of sgmmetry, our original problem is alo solved The reader will not fail to abserve that the second problem is more general than che fx, and, nevertheless, much easier than the Brit. In fact, oar main achievement in solving the frst problem was 10 invent the second [problen. Inventing tho second problem, we recognize the role of the center of symmety; we dsentangled that property of the octahodron which is essential for the problem at hand, samely that it has such a center The more general problem may be easier to solve. This sounds paracionical but, alter the foregoing example, it should not be paradoxical to us. The main achievement in solving the special problem was to invent the general problem, After the main achievement, only 4 minor pase ‘of the work remains. Thus, sn our ease, the solution of the general problem is only a minor part of the solution ‘of the special problem. See evens PARADOX, 4. "Find the volume of the frastumn of a pyramid with square bas, being given that zh side of che lower base is 104n., the side of the upper base g ., and the altitude Of the frustum 6 in.” IF for che numbers 19,5, 8 we |b tinue levers, for instance 2, b, h, we generalize. We ‘brain 2 more general problem than the original one, ‘namely the following: “ind the volume of the frustama 4, of a pyramid with square base, being given that the side [Ff the lower base is, the side of the upper base &, and so A Problem Related to Yours and Solved the alticude of the frustum A.” Such generalization may be very weful, Passing from a problem “in numbers” 10 nother one “in letter” we gain access to new proce- lures; we can vary the data, and, doing so, we may check four results ia various ways. See CAN YOU CHECK THE ESULE? 2, VARIATION OF THE PROBLEM, 4 Have you seen it hefore? It is possible that we have solved betote the sime problem that we have to do nos, for that we have heord of it, or that we bad a very similar problem. These are posibiliies which we should not fail to explore. We fry to remember wiat happened. Have you seen it Defore? Or have you seen the same throbs lem in a slightly diferent form? Even i€ the answer is negative such questions may start the mobilization of tuctul knowledge "The question in the title of the present article is often ‘used n'a more general meaning, in order o obtain. the solution, we have to extract relevant elements from ovr memory, we have co mobile the pestinent parts of oot dlormant knowledge (PROcRIAS aN ACHIEVEMENT). We fannot know, of couse, in advance which parts of our Knovladge may be relevant; but there are cercain po® sibilities whieh ye should not fail to explore. Thus, any feature of the present problem that played a role i the solution of some other problem may play again a role. ‘Therefore, if any feacure of the present problem strikes toy a8 possibly important, we 17 to reoogaize it. What is 4S Tse familiar to you? Have you seen tt before? Here is « problem related to yours and solved before. “This s good news; a problem for which the solution 3 known and which is connected with our prevent problem is certainly weleome, Iti tll moze welcome ifthe con festion is Close and the éolution simple. There isa good 4 Problem Related to Yours ond Solved —u14 ‘bance that such a problem will be useful in solving our resent one “The situation that we are discussing heve is typical and important. In order to see it clearly let us compare it ‘with the situation in which we find ourselves when we are working 2t an auxiliary problem. In both cases, our ‘aim is to solve a certsin problema a and we introduce and ‘comider another problem 2 in the hope that we may ‘derive some profit for the solution of the propated prob Jeo 4 trom the consideration of that other problem B. ‘The diference js in ur rslation to B. Here, we me ceeded in recollecting an old problem B of which we now the solution but wo do not know yet how to use it ‘There, we succeeded in inventing a new problem B: we know (or at least we suspect strongly) how to use By but wwe da not know yet how to solve it, Our dificuty co cerning B makes all the diference berween the two situ: ations: When this dificulty is avercome, we may wie 3 in the same way in both cases: we may use the result or the method (as explained in auxiiary Poste, 5), and, if wwe are lucky, we may ure both the recule and the method. In the situation considered here, we know well the sal tion of B but we do not know yet how to use it, There fore. we ak: Could you use i? Gould you use its result? Gould you uz its method? The intention of using a certain formerly solved prob Jem influences our conception of the present problecn ‘Trying to link wp the too problems, the neve and the ‘ald, we introduce into che new problem elements corre ‘ponding to certain important elements ofthe old prob. Jem. For example, our problem ir to determine the sphere circumscribed about a given tetrahedron. This is « problem of solid geometry. We may remember that we have solved before the anslogous problem of plane ‘geometry of constructing the circle rumscribed about 12 Hewiate ‘given trfangle. Then we recollect chat in the old prob lem of plane geometry, we used the perpendicular bie score of the sides of the triangle. Iti reavonable to ry to introduce something analogous into our presont probe lem. Thus we may be led to introduce into our prevent problem, as corresponding ausiliary clemenss, the per pendlicular bisecting planes of the edges of the tetra hodron, After chs idea, we can easly work out the solution to the problem of soll geometry, following the analogous solution in plane geometry. ‘The foregoing example is typical. The eonsideration cof a formerly solved relavod problem leads us to the introduction of auxiliary elements, and tbe introduction of suitable auxiliry elements makes it posible for us to tue the related problem :o full advantage in solving our present problem, We aim at such an effect when, think ing about the posible we of a formerly solved related problem, we ask: Should you introduce same avxiliary flement inorder to make its use possible? Hore is. theoters related to yours and proved before ‘This version ofthe remark discussed here is exemplified Simsection 19. Heuristic, or heuretic, or "ars inveniendi” was the name of a certain branch of study, not very clearly cr ‘cumscribed, Belonging to logic ot to philosophy, or 10 rychology, often outlined, seldom presented in detail, And as good as forgotien today. The aim of heuristic i study the methods and rules of discovery and invention. A fow traces of such study may be found in the com smentators of Euclid: « passage of pavecs is particularly interesting inthis respect. The tost famous attempts to Dad up a sytem of heuristic are due to oescarres and to ease, both great wathematicians and philosophers Berard sovzavo presented a notable detailed account of Heuristic Reasoning 7 heuristic. The preuent booklet is an attempt to revive Iheavistic ina modern and moslest form, See MoDEES curistic, as an adjective, means “serving to discover” ‘Heuristic reasoning is reasoning not regarded as Final and strict bue xs provisional and plausible only, whose purpose is to discover the solution of the present prot Jem. We are often obliged to use heuristic reasoning. We shall attain complete certainty when we shall have ob tained vhe complete solution, but before obtaining ce tainty we must often be satisfied with « mare or les plausible gues. We may need the provisional fare ‘we 2ttin the inal. We need heuristi reasoning when ve construct a strict proof as we need scaffolding when we erect a building See sons oF rxogness. Heuristic reasoning is often bbaed on induction, or on analogy: see NDUEHON AND MATHLMNTIEAL INDUEHON, and ANALOGY, 8, 10° Heuristic reasoning is good in itll. What is bad is fo mix up hevrietic reasoning with rigorous proof. What is worse isto sll heuristic reasoning for rigorous proof The teaching of certain subjects, especially the teach: 1g of calculus to engincers and physicist, could be enen tially improved if the nature of heuristic reasoning were Deer cniderstood, bot its advantages ana its fimieatins ‘openly recognized, and if the textbooks would present heuristic arguments openly. A heuristic argument pre sented with tase and frankness may he well; it may prepare for the rigorous argument of sthich it usually ‘omtxine certain germs. Bu + heuristic argument is lkely to be harms! fit is presented ambiguously with visible Wes YE wee tH Ameen Meet tag Induction and Mathematical Induction hesitation between shame and pretsion. See way ragors? Af you eannot solve the proposed problem do not let this flare affice yon too much but ty to find console tion with some essier sucess, try 10 solve frst sume res lated problem: then you may find courage to attack your original problem again. Do not forget that human super ‘ority consists in going around an obstacle that cannot be ‘overcome directly, in devising some suitable ausiliary problem when the original one appeas insoluble Gould you imagine a more accesible related problem? ‘You should now invent a related problem, noe merely remember one: I hope that you have tried already the ‘question: Do you know related problem? “Lhe remaining questions in that paragraph of the fist which starts with the tile of the presen: article have a ‘common aim, the VARIATION OF THE mORLEM. There are different means to attain this aim as crweRatizavion, SPECIALIZATION, AKALOGY, and others whi are various ways ol BuCONrOsNG AND RECOMBINE Indvetion god mathematical induction, Induction is the proces of discovering general laws by the observation and combination of particular instances. I i used io all sciences, even in mathematics. Mathematical induction {used in mathematics alone to prove theorems of certain kind. Iti rather unfortunate that the mazes are eoomectel bacause there i very lite logical connection ‘etwoen the two processes. There i however, some prac tical connection; we often use both methods together. We are going to illustrate both methods by the same example 1. We may abscrve, by chance, that 18427 +64 = 100 Induction and Mathematical Induction 45, and, recognising the cubes and the aquare, we may give to the face vee observed the more interesting form: 1 284334 48-20% How does such a thing happen? Does it often happen that sucle a sum of sueceisive cubes is 2 square? Tn asking thie we are Fike the naturalist who, impressed bby 2 carious plant ot a curious gcologieal formation, cn. ceives 4 general question. Our general question is cov. ‘zmed with che um of successive cubes seed Ege eet nt ‘We were Ted to it by the “particular inscance” n= 4 ‘What can we do for our question? What che naturalist ‘would do; we can investigate other special cases. The special cases = a, gare sill simpler, the ese 535 the next one. Let us add, forthe sake of uniformity and completeness, the case n= 1. Arranging neatly all these ceases, a5 a geologist would arrange his specimens of a ‘ert ove, we aban the follwing tale: 148 ans 14 8se7 = 36-6 148+ ep +64 — — 100m 108 DASE aT Bf m5 ~ 295 = 15% J ts hard to believe that all these sums of consecutive ‘cubes are squares by mere chance. In a similar case, the rast would have little doubt hat dhe general law suggested by the special caes heretofore observed i cor rect the general [ae is almost proved by mduction. The mathematician expresses himself with more reserve a: though fundamentally, of course, he thinks in the same fashion, He would say that the following theorem. is stongly suggested by induction: “The suns of the fist weibes ia aqua, 136 Induction end Mathemesical Induction 2, We have been Jed to conjecture a remarkable, some: ‘what mysterious law. Why should those sums of succes: sive cubes be squares? But, apparently, they are squares. ‘What would the naturalist do in such a siuasion? He ‘would go on examining his conjecture. In 30 doing, be say follow various tines of investigation, The naturalist may accmlate further experimental evidence: if we ‘wish 10 do the sume, we have to test the next cases n= 67, Phe naturalist may ako reexamine the faci where observation has led him «o his conjecture, he compares them carefully, he tics to disentangle some deeper tegularity, some further analogy. Let us follow this ine of investigation. Let us reexamine the cases n=, 2, 3, 45 which we arranged in our table, Why are all these sums squares? ‘What ean we say about these squares? Their bases ere 1 43:6, 10, 15, Weat about theve bes? Is ehere some deeper egulavty, some farther analegy? Atany rate, they do not soem to increase too irzegulatly. How do they increase? ‘The diference between two successive terms of this se quence is ise inereasing, 8 mm 8 gng 0-8 eh 5-H Now these differences are conspicuously regula. We may see here # surprising analogy Between the bases of chose faqusres, we may gee a remarkable regularity i ce ume bere 1, 3 6 20,15 grite Om itets. rostben ste rare tats Uf ehis regularity is general (and the contrary s hard «0 Induction and Mathematical Induction — 129 believe) the theorem we suspected takes a more precise form: This for n= 8 § bag ghee nd a (at gered mp 4 The law we just stated was found by induction, and the manner in which it was found conveys to us an idea bout induction which is necessarily onesided and im- perfect but not distorced. Induction tries to find regulare ity and coherence behind the observations. Its most con- spieuous instruments are generalization, specialization, analogy. Tentative generalization starts from aa effort t0 lunderstind the observed Exes; its baed om analogy, 2d tested by further special cases. ‘We refrain from further remarks on the subject of fnduction about which there is wide disagreement among philosophers. But it should be added that many mathe: ‘matical results were found by induction first and proved Inter. Mathematics presented with rigor is a systematic deductive science but mathemades in the making is am ‘experimental inductive xience ‘Ta mathematics as ia che physeal sciences we may use observation and induction to discover general laws Bur there isa difference. Inthe physical sciences, there i ro higher authority than observation and induction bt Jn mathematics there is sch an authority: rigorous roof. ‘After having worked a while experimentally it may be ‘good to change our point of view. Let us be stir. We Ihave discovered an interesting result but the reasoning that led to it was merely plausible, experioretal, prov ‘isional, heuristic; let us Wy to establish it definitively by a rigorous proof We have arrived mow at 2 “problem to prove": to hiB Induction and Mathemetical Induction prove or to disprove the rele stated before (see 2 above) “There is 8 minor simplification. We may know that itetgsee $a 0k Avany rate his is easy 10 verify. Take a rectangle with sides ard m 4-1 and divide it in two halves by a zigzag line as in Big. 152 which shows the case n= 4. Fac of the halves is “saireaseshaped” and its atea Bs the ex: presi percrbn form=4 it rbot gt g, see Fig. v8, Now, the whole area of the rectangle is niu +1) of whieh the staircaveshaped area is one hell this proves the formula, s ne. 8 We may transform the result which we found by im Auction into Pattee +e (eeu 5. Tee have no idea how to prove this result, we may at Teast test it, Let us text the fret case We fave not fevted yet, the cate m= 6, For this value, the formule vieles Debt ar beet ey sa = (IY Induction and Mathessatical Induction 149 nd, on computation, this turns eut to be true, both sides being equal to 449 ‘We can tat che formula more efeeively. The formula is, very likely, generally cue, uve for all salves ofr, Does it remain irue when we pas from any value » to the next value m+ 17 Along with the formula as written above (p. 118) we should alo have SEES Hedger or (CEMeAY. Now, chose i simple check. Subwacting from this the formuis weiten above, we obtain eon (GtDBEDY _ (oboy, ‘This is, however, easy to eck. The right hand side may be writen as CB tat te LY beta WEP peg mroter d= + (ur experimentally found formula pased & vital text Let us see clearly what thi test means. We verified beyond doube that fore (Ctnetay (uetay, ‘We do not know yer whecher Vette petee is crue. But if we knew tbat this was true we could infer, Dy adding the equation which we verified beyond doubt, that seboy Pbategthectatetehiy= (@bmetay t29 Induction and Mothematical Induction fs also etue which is the same asertion for the next Integer n+ 1, Nov, we actually know that our conjec rue is ue for 1 ="1, 2, § 4 5 By virtue of what we dave just sul, che conjecture, being true for n ~ 6, must also be true for n = 7; being true lor n ~ 7 its trve for n= 8; being true for n ~ 8 itis rue for m ~ 9; and so ‘on. It holds for all n, iis prowed to be true generally. 5. The foregoing proof may verve as 2 potlern in many similar cases. What are tne essentialines of thie pattern? ‘The anertion we have to prove must be give vance, in precise form. "The assertion must depend om ay The assertion must be suficiently “explicit” so that we have some posibility of vesting whevher ic remaias true in che passage from n to Une next integer 2 +1 1 we succeed in esting this eflectvely, we may be able to use our experionce, guined in the process of testing, to conclude that the asiertion must be true for n+ pro- vided i is true for n. When we are so far i is suficient to Know thatthe asertion is true for = 15 hence it fallosee for n= hence it follows for n ~ 3, and s0 om; passing from any integer to the next, we prove the assertion generally. This process is xo often wed that it deserves a nome. ‘We could cal 1“ proot from 1 10 2 + +" stil simpler passage to the next integer.” Unfortunately, the a ‘cepted technical term is “mathematieal induction." This ame results from a random circumstance, The precise aseertion thc we have to prove may come from any fouree, and itis immaterial from the logieal viewpoint what the source is. Now, fa many cass, as ia the eate we discussed here in detail, the source is induction, the ser tion is found experimentally, and s0 the proof appears ae a mathematical complement 10 induction; this ex plain che name. Inventor's Poredos a ‘i Here is another point, somewhat subee, but impo tant to anybody who desires t9 Bul proofs by himets In the foregoing, we found two different assertions by ‘observation and induction, one after dhe oer, the Bret under 1, che second under 2; the second was more pre ‘se than the fist. Dealing with the second assertion, we found 2 possibilty of checking the pasage from n to ++ and so we were able to ind a proot by “matberate feal induction” Dealing with the fst assertion, and jaring the precision added to it by the second one, we should scarcely have been able to find seh a proot In fact, the firs asrtion i less preci les "explicit" ss gible,” less accesible to testing and checking than the second one. Passing from the Hrs to te sccond, from the less precise (o the more precise satement, was an important preparative for the Knal proot. ‘This circumstance has a paradoxical aspect. ‘The second assertion is stronger; i€ implies immediately the nt, whereas the somewhat "hazy" fist asertion ean hardly imply the more “clearcut” second one, Thus, the seronger theorem is exsier to master than the weaker (one this the LwVENOR's PARADOX. Tnventor’s paradax, The more arabitious plan may Ihave more chances of success This sounds paradoxiea. Yet, when passing from one problem to another. we may often observe thar the new, ‘more ambitious problem is easier so handle than the original problem. More questions may be easier 10 an- swer tam just one question. ‘The more comprehensive theorem may be easier to prove, the more general prob- lem may be easier to solve, “The paradox dhappeam if we lok coe at 2 few ‘examples (GENERALIZATION, 2: DUCTION AND MATHESCAT: texs INouCHION, 7). The more ambitiows plan may kave la Is Lt Ponible to Satiaty the Condition? more chances of success provided it fs not based on mere pretension but on seme vision of the dings beyond those Jmmediately prevent 1s it possible to satiaty the condition? Is the condition suficient to determine the unknownd Oris it innwfictent? Or redurdant? Or contradictory? ‘These questions are often useful a¢ an eary tage when. they do not needa fal answer but just 2 provisional answer, 4 guess. For examples, see sections & 18. Tis good to foresee any feature of the rest for which ‘we work, When we have some kea of what we can ex: pect we Know better in waich direction we should go. Now, am impoctane feature of a problem is the number of solutions of which it admits. Most interesting among problems are those which admit of just one solution; we te inclined to consider problems with a uniquely dev fhined solution asthe only “reasonable” problems. 1s our problem, in this sense, “easonable”? Hf we can answer this question, even by a plausible gues, our interest in the problem increases and we can work better {sour problem "rewonable”? This question is useful acan early sage of our work if we can ansver it exsily. If the answer ib dificult to obtain, the trouble we have jn obtaining it may ouzweigh the gain in interes, The fame is true of the question “Is it posible to satisfy the Condition?” and the allied questions of our ist. We Should put them became the answer might be exyy and plausible, but we should not insist om Chem when the nswer seems to be digicult or obscure. "The corresponding questions for “problems t0 prove” ane: Ist likely thatthe proposition is true? Oris it more Tikely that itis flac? The way the question is pus shows cleerly that only a gues, a plausible provisional answer, in expected. i Look at the Unknown 3 ried Wilhelm (1646-1738), great mathe iatician and philosopher, planned to weite an "Art of Invention” but he never carried through his plan. Nu rerous fragments dispersed. in his works show, how. fever, tha He entertained jatersting ideas about the fubject whose importance he often emphasized. ‘Thus, he wrote: “Nothing is more important than to ste the source af invention which are, in my opinion. moe jmeresting than the inventions themselves.” ‘Lemma means “auxiliary theorem," The word is of Greck origins a more literal translation would be “what ieassumed.” ‘We are tying ta prave a theorem, say, . We ate led to suspect another Cheorem, say, IC B were true we could ‘perhaps, csing it, prove 4. We assume B provisionally, pportponing its proof, and go ahead with the proof of ‘Sach a cheocer is asumed, and is an auxiliary theorem to the originally proposed theorem 4. Our lete story is fuily typical and explains che present meaning of the Look at the unknoven. ‘This is old advice: the corre- sponding Latin saying fst “respice nem.” That i look atthe end. Remember your aim. Do not forget your goal ‘Think of what you are desing to obiain. Do not lose sight of what ie required. Keep in mind what you are working for. Look at the unkncun Look at the conclu sion. The last two versions of “respice finem” are spe- ‘cifeally adapted w siathematcal problems, to “problems to find” and to “problems to prove” respectively. Focusing our attention on our aim and concentrating ‘our will on our purpose, we think of ways and means © attain it, What ate the mneans to this end? How can you. attain your aim? How can you obtain a result of thie 124 Look at the Unknown lode What coues could produce uch a zen? Where ve you sen mich # ret prodaced? What do people Cou dt oti ach ev nd yf think of tame problem lacing the sae Or stlar enhnown. "ie ty to hn of amin thorem having the sme 2 abun conclusion, ain he Ia to Yerion ae ‘peneally adapud to robles to Bad” and to “pro Jens ope” recive Ware ping to comer mathematical prebens spoblems to hee andthe soggestion. Try think of a instr problem hong te seme unkown, Let ws e- fare ts gaan With tat five the question Doyen know 6 ead problen “The intr sggeon ne general than the former ce Ita probe is elated to another problem, te two fave something i oatn, they say Bale few com thom objets of non or haves dal is combo, or Ste part of te conden, and 99 on. Our fs ge {lon ipsa on 2 pela common point The two problems should have theme unknown, That i the Erkown stn be i Both css abet ofthe same Cctegorys for fsa, ia bes caves he length of Seaigu line Titcomparion wih the geotral aogeion, dee sa certain economy in the special uggeaion. First we may sate ome elo represen the probe emt oe mus tot lo at once tthe whol problem bat festat Oe unknown, The problem appears to ve schma- realy “Given find the length of the Tne.” Second, there is x certain economy of choice, Many, many problems may be related to the proposed problems, having some point oF other in common with it, Bute looking at the unknown, we restrict our choices we take Look at the Unknown 185 {nwo consideration only such problem 2¢ have the some ‘unknown. And, of course, among the problems having the same unknown, we consider fst those which are the ‘most elementary and the most familiar 9 us. ‘2 The problem before us has the frm [Now the simplest and mse familar problems of thi Kind are concemed with triangles: Given three consie vuent parts of a triangle find the length of a side. Re- iembering this, we have found something that may be relevant: Here a problem related ¢o yours and solved before. Could you awe i Could you use ite resull kn ‘order to use the Familiar results about triangles, we must hhave a triangle in our figure. Is there a triangle? Or should we introduce one in aidee to profit From hove familiar remlu? Should yow introduce some ausiliary element in order to make their wie possible? “There are several simple problems whose unknoven is the side of triangle. (They difer from each othe inthe data; two angles may be given and one side, of two sides tnd one angle, andthe postion of the angle with respeet to the given sides may be diferent. ‘Then, all these prob- Jen are particularly simple for right triangles) With ‘our attention riveted upon the problem before ut, we ty to find out which kind of triangle we should introduce which formerly solved problem (with dhe sme vnknown as that before us) we eould mest conveniently adapt to ‘our present parpore ‘iaving introduced a suitable auxiliary triangle, it may Dhappen that we do not know yet three constituent parts ‘of it This, however, is not absolucely necessary; If we oresee that the missing parts can be obtained somehow ‘we have made essential progrest, we have a plan of the solution, find the fength ofthe line.”

Вам также может понравиться